SlideShare a Scribd company logo
1 of 200
Download to read offline
Exam Section 1: Item 1 of 50 National Board of Medical Examiners^
Comprehensive Basic Science Self-Assessment
Y 1. A 25-year-old man is brought to the emergency department because of severe abdominal pain, nausea: and vomiting for 1 hour. The pain originates in the left flank and radiates to his groin. His pulse is 100/min, respirations are 18/minp and blood pressure is
150/100 mm Hg. Physical examination shows tenderness of the left flank and the left lower quadrant of the abdomen. Bowel sounds are mildly hypoactive. Test of the stool for occult blood is negative. Which of the following best explains these findings?
A ) Colon neoplasm
B) Diverticulitis
C ) Epididymitis
D ) Renal infarction
E) Torsion of the testis
F) Ureteral calculus
Correct Answer: F.
Ureteral calculus typically presents with colicky unilateral flank pain radiating to the groin: and with gross or microscopic hematuria. Pain may be significant enough to trigger nausea, as in this case. The common types of urinary tract calculi are calcium oxalate or
phosphate, ammonium magnesium phosphate: uric acid: and cystine. On urinalysis, red blood cells without casts are common. Fever dysuria; and pyuria would not be expected unless there was a concomitant infection. Treatment for ureteral calculus is
symptomatic with pain control and nausea relief. Most ureteral calculi pass spontaneously after a period of observation for patients with well-control ed pain and no signs of sepsis or infection. Stone removal by shock wave ithotripsy or endoscopic removal is an
option for patients requiring emergency therapy. It is also an option for patients with persistent obstruction, uncontrolled symptoms, or failure of stone progression. In general, stones smaller than 5 mm will pass without operative assistance. Obstructing stones may
require temporary placement of a ureteral stent to prevent hydronephrosis and renal parenchymal injury.
Incorrect Answers: A. B. Ca D, and E
Colon neoplasm (Choice A) would be unlikely in an otherwise healthy young patient with no family history of polyposis syndromes and acute, severe, flank pain. It would typically present with insidious weight loss, anemia, constipation or blood per rectum. In
addition, test for stool for occult blood is negative, making this diagnosis unlikely.
Diverticulitis (Choice B) can present with left lower quadrant abdominal pain and tenderness but would be less abrupt in presentation and typically present with fever, diarrhea, and hyperactive bowel sounds. It would be unlikely to cause flank pair
Epididymitis (Choice C) is a common cause of painful scrotal swelling and refers to acute infection and inflammation of the epididymis. In younger males, this is commonly secondary to sexually transmitted infections such as Chlamydia trachomatis or Neisseria
gonorrhoeae. In older males, Escherichia coli is more common.
Renai infarction (Choice D) can cause flank pain, nausea, and vomiting, and can be due to thromboembolic disease, renal artery dissection, or a hypercoagulable state. However, it is rare and ureteral calculus is mare common and likely in this patient.
Torsion of the testis (Choice E) occurs when the testicle twists on the spermatic cord resulting in subsequent loss of testicular blood supply. Patients typically present with acute, severe testicular pain, swelling, and erythema. On physical examination, the testicle
typically demonstrates an abnormal lie (eg, transverse), extreme tenderness to palpation, absent cremasteric reflex, and pain that does not improve with elevation of the scrotum (as it does in epididymitis).
Educational Objective: Ureteral calculus typically presents with colicky, unilateral flank pain radiating to the groin, along with gross or microscopic hematuria.
0 0 <t= A
Next Score Report Lab Values Calculator Help Pause
Exam Section 1: Item 2 of 50 National Board of Medical Examiners^
Comprehensive Basic Science Self-Assessment
Y
2. Which of the following types of sensory information is compromised by lesions of the structure at site X in the photograph shown?
A } Conscious proprioception
B) Pain sensation
C) Two-point discrimination
D) Unconscious proprioception
E) Vibration sense
Correct Answer: D.
The anterior lobe of the cerebellum (labeled X. pictured in cross-section as an arborized brain area posterior to the brainstem and anterior to the primary fissure of the cerebellum) mediates unconscious proprioception. The anterior lobe of the cerebellum receives
information from the spinocerebellar tract about proprioception, or body position, that is gathered from muscle stretch and tension receptors on the ipsilatera! side of the body. This proprioceptive information is transmitted outside of conscious awareness. The deep
cerebellar nuclei use this proprioceptive information to control motor learning, movement course changes, and balance. Damage to the anterior lobe of the cerebellum, which commonly occurs in chronic alcoholism, may lead to broad-based gait ataxia.
Incorrect Answers: A. B: C. and E.
Conscious proprioception (Choice A), two-point discrimination (Choice C), and vibration sense (Choice E) are mediated by the dorsal column-media lemniscus pathway, which relays this sensory information up the spinal cord to the thalamus and terminates in the
primary sensory cortex in the parietal lobe. The cortex is a high-order brain area involved in several conscious brain functions, which reflects this pathway's mediation of the conscious (rather than unconscious) awareness of proprioception.
Pain sensation (Choice B) is mediated by the spinothalamic pathway. The spinothalamic pathway transmits information about pain, temperature, and crude touch up the spina! cord to the thalamus, terminating in the primary sensory cortex.
Educational Objective: The anterior lobe of the cerebellum mediates unconscious proprioception, whereas conscious proprioception is controlled by the dorsal column-medial lemniscus pathway. Lesions of the anterior lobe of the cerebellum can result in broad-
based gait ataxia.
0 0 0
Previous Next Score Report Lab Values Calculator Help Pause
Exam Section 1: Item 3 of 50 National Board of Medical Examiners^
Comprehensive Basic Science Self-Assessment
Y 3. Moving the forearm against resistance from palm-down to palm-up (supination) position requires the use of which of the following muscles?
A) Biceps brachii
B) Brachialis
C) Triceps
D) Flexor carpi radialis
E) Pronator teres
Correct Answer: A.
The biceps brachii muscle has two main actions, flexion of the elbow joint and supination of the forearm. The biceps brachii contains two proximal heads, with the short head attaching to the coracoid process of the scapula and the long head entering the shoulder
joint and attaching to the supraglenoid tubercle. The distal biceps tendon inserts on the bicipital tuberosity of the proximal radius. Because of its orientation crossing the elbow joint, contraction of this muscle causes elbow flexion. Its eccentric insertion on the
proximal radius allows for it to wind around the radius during pronation and unwind when contracted from around the proximal radius during supination.
Incorrect Answers: B, C. D and E
The brachialis muscle (Choice B) originates on the anterior surface of the humerus and crosses the elbow inserting on the tuberosity of the ulna. It does not wrap around the ulna and the ulna does not rotate. Because of this, it does not contribute to supination or
pronation.
The triceps muscle (Choice C) serves to extend the elbow joint. Proximally, it originates from the infraglenoid tubercle of the scapula (long head}, just proximal to the radial groove (lateral head), and just distal to the radial groove (medial head). Distally: it inserts on
the olecranon process of the ulna. Contraction of this muscle extends the elbow and does not contribute to rotation.
Mexor carpi radialis (Choice D) originates on the medial epicondyle of the humerus and inserts on the second and third metacarpal bones. This allows for flexion of the wrist.
Pronator teres (Choice E) is a muscle of the proximal forearm that extends from the medial supracondylar ridge of the humerus and inserts on the lateral aspect of the radius. Contraction along this axis will promote pronation: not supination.
Educational Objective: The biceps brachii muscle has two main functions. It serves to supinate the forearm through its winding mechanism around the proximal radius and bicipital tuberosity. It flexes the elbow as the muscle and tendon cross the elbow joint
anteriorly.
0 0 & 0 0
Previous Next Score Report Lab Values Calculator Help Pause
Exam Section 1: Item 4 of 50 National Board of Medical Examiners^
Comprehensive Basic Science Self-Assessment
Y 4. A 35-year-old man is admitted to the hospita because of a 5-day history of fever and dyspnea He underwent a bone marrow transplantation S months ago; the procedure was complicated by severe graft-versus-host disease. His temperature is 33°C
(100.4T) and respirations are 30/min. Scattered crackles are heard on auscultation of the chest. A chest x-ray shows patchy infiltrates A transbronchial biopsy specimen shows findings consistent with cytomegalovirus infection. Intravenous administration of
ganciclovir is begun. This drug interferes with the function of which of the following enzymes?
A) DNA polymerase
B) Jntegrase
C ) Reverse transcriptase
D ) RNA polymerase
E) Thymidine kinase
Correct Answer: A.
Cytomegalovirus (CMV), also known as human herpesvirus 5 (HHV-5); is an opportunistic infection commonly occurring in immunocompromised patients with solid-organ or allogeneic bone marrow transplantation severe ulcerative colitis, or HIV/AIDS infection. It
can be transmitted through multiple modes, including sexuai contact urine, respiratory droplets: and to a fetus via the placenta. It can cause a variety of presentations, including mononucleosis in immunocompetent patients, and retinitis, esophagitis, and pneumonia
in immunocompromised patients. Treatment for all human herpesvirus infections involves drugs that inhibit viral DNA polymerase, classically by guanosine analogs such as acyclovir valacyclovir, and famciclovir. Prior to exerting their antivira effects, mos'
guanosine analogs must be phosphorytated by the viral enzyme thymidine kinase. They are then able to inhibit the viral DNA polymerase by terminating the nascent DNA chain during replication. These drugs are effective against herpes simplex virus and varicella
zoster virus, weakly effective against Epstein-Barr virus, and not effective against CMV, which does not have the necessary thymidine kinase needed for phosphorylation. However, it does have the necessary phosphorylating enzyme to activate ganciclovir, another
guanosine analog that inhibits DNA polymerase, and thus this is an effective anti-viral treatment for CMV
Incorrect Answers: B. C, D, and E.
Integrase inhibitors (Choice B) prevent integration of proviral DNA into the host genome. They are a component of the highly active anti-retrovirai therapy utilized in the treatment of HIV.
Reverse transcriptase (Choice C) transcribes DNA from viral mRNA for incorporation into the host cell genome. This enzyme is inhibited by both nucleoside reverse transcriptase inhibitors, such as abacavir and didanosine. and non-nucleoside reverse transcriptase
inhibitors, such as efavirenz and nevirapine. These medications are used to treat HIV infections.
RNA polymerase (Choice D) is the enzyme that facilitates the conversion of DNA to RNA. It is the target of nucleotide and nucleoside polymerase inhibitors, which are utilized in the treatment of Hepatitis C infection.
Thymidine kinase (Choice E) is a phosphorylating enzyme required for activation of the guanosine analogs acyclovir valacyclovir. and famciclovir. Development of a mutation in the viral thymidine kinase enzyme prevents drug phosphorylation and confers
resistance to these medications. CMV does not contain the thymidine kinase enzyme.
Educational Objective: Ganciclovir is a guanosine analog that inhibits DNA polymerase and is used to treat CMV
. Like other guanosine analogs activation of this medication requires phosphorylation within the infected cell
© © ©
Previous Next Score Report Lab Values Calculator Help Pause
Exam Section 1: Item 5 of 50 National Board of Metlicai Examiners^
Comprehensive Basic Science Self-Assessment
y
0
200-
T3
rS E
E CL 100-
<t>
c/>
0
25 27 1
Cycle (day)
M = menstruation
5. The graph shows changes in serum estradiol concentration during a normal menstrual cycle. Which of the following ovarian cells is primarily responsible for the aromatization of androstenedione to estradiol at the time indicated by the arrow?
A) Granulosa
B) Luteal
G) Stromal
D) Theca externa
E) Theca interna
CorrectAnswer A.
The first half of the menstrual cycle; the follicular phase: which varies in length, begins with menses. During menses, follicle-stimulating hormone (FSH) and luteinizing hormone (LH) concentrations increase and stimulate the developing follicle. Androstenedione
is converted to estrone and estradiol via aromatase in the granulosa cells of the follicle. The estrogen then secreted from the granulosa cell is responsible for follicle growth and endometrial proliferation. As estrogen rises, a surge occurs, which in turn stimulates a
surge in LH that causes ovulation. Immediately following ovulation, the luteal phase begins as the corpus luteum forms. The corpus luteum secretes progesterone to maintain the endometrial lining. However if no implantation occurs, the corpus luteum degrades
to the corpus albicans, and estrogen and progesterone levels decrease, causing menstruation and minor increases in FSH and LH.
Incorrect Answers B, C, D, and E.
Luteal cells (Choice B) are present in the corpus luteum and are derived from the granulosa cells of the pre-ovu atory follicle. They secrete progesterone and estrogen. However they do not develop until after ovulation (14 days prior to menstruation).
Stromal cells (Choice C) are the connective tissue and supporting cells of the ovary. They do not secrete estradiol.
Theca externa cells (Choice D) are the cells that form the outer layer of a developing follicle. The theca externa is primarily loose connective tissue and therefore the cells are generally fibroblasts, macrophages, and smooth muscle; these cells do not secrete
hormones.
Theca interna cells (Choice E) are cells of the follicle that are responsible for generating androstenedione from cholesterol, after which the androstenedione is transported to the granulosa cell to be converted to estradiol.
Educational Objective: Granu- osa cells in the developing follicle are responsible for converting androgens received from the theca interna ceils into estradiol via aromatase.
e
Previous Next Score Report Lab Values Calculator Help Pause
Exam Section 1: Item 6 of 50 National Board of Medical Examiners^
Comprehensive Basic Science Self-Assessment
Y 6. A 14-year-old girl is brought to the physician by her parents because of a 1-month history of a rash that appears with sun exposure. Her parents tell the physician that she has been eating little food. Physical examination shows a pruritic rash on the exposed
areas of the body. Her serum tryptophan concentration is decreased. Urine studies show increased excretion of amino acids, predominantly alanine: isoleucine: leucine, phenylalanine, tryptophan, and valine. Production of which of the following vitamins is
most likely impaired in this patient?
A) Niacin
B) Vitamin B1 (thiamine)
C ) Vitamin B2 (riboflavin)
D } Vitamin B5 (pantothenic acid)
E) Vitamin C
Correct Answer: A.
Hartnup disease is an autosomal recessive disorder involving a defect in a kidney and intestinal neutral amino acid transporter protein. This defect leads to aminoaciduria and a decreased absorption of neutral amino acids from the gastrointestinal tract resulting in
deficiencies of neutral amino acids. Neutral amino acids include tryptophan, phenylalanine, glycine, alanine, vafine: isoleucine, eucine: methionine, and proline. Tryptophan is converted to niacin, so a deficiency in tryptophan can result in niacin deficiency. Niacin
deficiency is characterized by rash, glossitis, diarrhea, and neuropsychological disturbances such as dementia and hallucinations.
Incorrect Answers: B, C, D, and E.
Vitamin B-| (thiamine) (Choice B) is a cofactor for several enzymes in glucose metabolism and adenosine triphosphate production, including pyruvate dehydrogenase and a-ketoglutarate dehydrogenase. Deficiency is characterized by Wernicke encephalopathy, a
triad of confusion, ophthalmoplegia, and ataxia. Wernicke encephalopathy is theoretically reversible with administration of high-dose thiamine; if untreated, it can progress to Korsakoff syndrome which is characterized by dementia, confabulation, hallucinations, and
psychosis.
Vitamin B2 (riboflavin) (Choice C) deficiency is characterized by inflammation and cracking of skin around the lips, mouth, and tongue. It is not associated with aminoaciduria or Hartnup disease
Vitamin B5 (pantothenic acid) (Choice D) deficiency is characterized by dermatitis, enteritis, alopecia, and adrenal insufficiency, it is not associated with aminoaciduria or Haitnup disease
Vitamin C (Choice E) is found in fruits and vegetables and is necessary far collagen synthesis, iron absorption, immune function and conversion of dopamine to norepinephrine. Deficiency causes scurvy, which is characterized by swollen gums, bruising and poor
wound healing, petechiae, perifollicular and subperiosteal hemorrhages, and short, fragile, curly hair.
Educational Objective: Hartnup disease is an autosomal recessive disorder involving a defect in a kidney and intestinal neutral amino acid transporter, leading to deficiencies in neutral amino acids such as tryptophan. Tryptophan is converted to niacin, so a
deficiency in tryptophan can result in niacin deficiency. Niacin deficiency is characterized by rash, glossitis, diarrhea, and neuropsychological disturbances such as dementia and hallucinations.
It
Previous Next Score Report Lab Values Calculator Help Pause
Exam Section t: Item 7 of 50 National Board of Medical Examiners^
Comprehensive Basic Science Self-Assessment
Y 7. Aminoglycoside antibiotics are used for their synergistic action against bacteria, in combination with other agents. These antibiotics demonstrate in vitro synergy for several bacterial species when combined with which of the following classes of antibiotics?
A ) Fluoroquinolones
B) Macrolides
C ) Penicillins
D ) Rifamycins
E) Tetracyclines
Correct Answer: C.
Aminoglycosides include gentamicin, neomycin, amikacin, tobramycin: and streptomycin. Their bactericidal function comes from the inhibition of the 3GS subunit of the bacterial ribosome, which precludes protein synthesis by causing misreading of mRNA.
Aminoglycosides work synergistically with penicillins, meaning that the combined effect of the two classes is stronger than the effect of either class alone. The penicillin group of antibiotics includes penicillinase-sensitive penicillins (penicillin G; ampicillin, amoxicillin)
penicillinase-resistant penicillins (oxacillin, nafcHlin, dicEoxaciflin), and anti-pseudomonal penicillins (ticarcillin, piperacillin). These all inhibit peptidoglycan cross-Einking of the bacterial wall. Aminoglycosides also demonstrate synergistic activity with monobactams
such as aztreonam, which also target peptidoglycan cross-linking function. However they do require oxygen for their uptake info the bacterial cell so are ineffective against anaerobes. Bacteria may also develop resistance 1o this class of antibiotics due to the
inactivation of the drug by bacterial transferase enzymes, which slightly modify the aminoglycoside structure through acetylation or phosphorylation. Aminoglycoside usage may be complicated by nephrotoxicity, ototoxicity, or neuromuscular damage. They should
not be used during pregnancy as they are also a teratogen.
Incorrect Answers: A. B( D. and E.
Fluoroquinolones {Choice A) inciude ciprofloxacin, levofloxacin, and moxifloxacin. They inhibit prokaryotic DNAgyrase (also known as topoisomerase). Potential side effects include vascular damage, cartilage damage, tendonitis, or tendon rupture. They do network
synergistically with aminoglycosides.
Macrolides (Choice B) include antibiotics such as azithromycin and erythromycin. These antibiotics also inhibit protein synthesis but instead by inhibiting the SOS subunit of the ribosome. Use of macrolides with aminoglycosides would not provide additional
therapeutic benefit particularly given that these classes of drugs have similar mechanisms of action.
Rifamycins (Choice D) inhibit the RNA polymerase required to transcribe bacterial DMA. They do not lead to synergistic effects when used with aminoglycosides.
Tetracyclines (Choice E) include tetracycline, doxycycline, and minocycline. Like aminoglycosides, these interfere with the 30S subunit of the ribosome. However, rather than causing misreading of mRNA, they prevent attachment of the aminoacyl-tRNA. They do
not lead to a synergistic effect when used with aminoglycosides.
Educational Objective: Aminoglycosides include gentamicin, neomycin, amikacin, tobramycin, and streptomycin. They inhibit the 3DS subunit of the bacterial ribosome. When used with peptidoglycan linking antibiotics such as the penicillin class, they lead to a
synergistic effect and result in improved bacterial killing.
© © © © f*
Previous Next Score Report Lab Values Calculator Help Pause
Exam Section 1: Item 8 of 50 National Board of Medical Examiners^
Comprehensive Basic Science Self-Assessment
8. A 16-year-old boy with moderate intellectual disability is brought to the physician for a routine examination. There is a family history of mild and moderate intellectual disability in his mother and brother, respectively. Physical examination shows a long face
prominent ears, and moderately enlarged testicles. Which of the following best describes the genetic mechanism of this patient's disorder?
A ) Mutation in a mitochondrial gene
B) Presence of an extra sex chromosome
C) Translocation of a portion of an aufosome
D) Trinucleotide repeat mutation on theX chromosome
E) Trisomy of an autosome
Correct Answer: D.
This patients constellation of findings, including intellectual disab: rty, an elongated face: large ears: and macro-orchidism is suggestive of Fragile X syndrome. Patients may also present with hyperextensible joints and a high-arched palate. Fragile X syndrome is a
common cause of inherited intellectual disability and is inherited an X-linked dominant fashion. It is caused by a CGG-trinucleotide repeat expansion within the FMR1 gene. Patients with Fragile X syndrome are at increased risk of mitral valve prolapse and
educational difficulties. They are also often diagnosed with autism. As with all trinucleotide repeat expansions, genetic anticipation is seen, wherein future generations have increased severity and/or earlier onset of disease.
Incorrect Answers: A, B. C, and E.
Mutation in a mitochondrial gene (Choice A) is observed in neurologic and muscular diseases such as Leber hereditary optic neuropathy, maternally inherited diabetes and deafness. myodonic epilepsy with ragged red fibers, and mitochondria! encephalopathy,
lactic acidosis, and stroke-like episodes (MELAS) syndrome.
Presence of an extra sex chromosome {Choice B) s observed in Klinefelter syndrome, in which patients have an XXY karyotype. Characteristic features include intellectual disability, eunuchoid body shape, tall stature, elongated extremities, and hypogonadism.
Translocation of a portion of an autosome (Choice C) is observed in Robertsonian translocations and causes a small portion of cases of Down syndrome. Characteristic physical findings of Down syndrome include intellectual disability, broad, flat, facial features with
prominent epicanthal folds, and a single palmar crease. Patients with Down syndrome are at increased risk of Alzheimer dementia, acute lymphoblastic and acute myeloid leukemia, cardiac septal defects, duodenal atresia, and Hirschsprung disease.
Trisomy of an autosome (Choice E) is observed in Down syndrome (trisomy 21), Edwards syndrome (trisomy 18), and Patau syndrome (trisomy 13). Edwards syndrome presents with characteristic physical features including intellectual disability, a prominent
occiput, low-set ears, micrognathia, clenched, overlapping fingers, and feet with a prominent calcaneus and convexly rounded soles. Patau syndrome presents with characteristic physical features including intellectual disability, deft lip and palate,
holoprosencephaly, microphthalmia, cutis aplasia, feet with a prominent calcaneus and convexly rounded soles, and polydactyly.
Educational Objective: Fragile X syndrome is an X- inked dominant cause of inherited intellectual disability that presents due to a CGG-trinucleotide repeat expansion in the FMR1 gene, it presents with clinical features including intellectual disability, an elongated
face with a prominent jaw. a high-arched palate, large ears, hyperextensible joints, and postpubertal macro-orchidism.
© © © 0* &
Previous Next Score Report Lab Values Calculator Help Pause
Exam Section 1: Item 9 of 50 National Board of Medical Examiners^
Comprehensive Basic Science Self-Assessment
Y 9. A 75-year-old woman comes to the physician because of a 3-month history of an enlarging lesion on her face. Physical examination shows a 1.5-cm, brown-black, mottled, scaly lesion with irregular borders. Microscopic examination of a biopsy specimen of
the lesion shows atypical melanocytes spread along the basilar layer of the epidermis. Which of the following is the most Eikefy cause of these findings?
A ) Acanthosis nigricans
B) Actinic keratosis
C ) Compound nevus
D) Lentigo maligna
E) Seborrheic keratosis
Correct Answer: D.
Malignant melanoma is likely to be present when a lesion demonstrates asymmetry, irregular appearing borders, variable coloration, a diameter greater than 6 mm, and rapid evolution in characteristics. Malignant melanoma could rapidly invade and metastasize,
which carries a poor prognosis when diagnosed late. Subtypes include superficial spreading, nodular, lentigo maligna, and acral lentiginous The lentigo maligna type is classically seen in elderly individuals in areas of extensive sun damage, such as the face.
Lentigo maligna typically grows slowly and superficial along the dermal-epidermal junction. Any lesion with features suggestive of malignant melanoma should be surgically excised with negative margins and pathologically examined for the depth of derma
*
invasion.
Incorrect Answers: A. B, C. and E.
Acanthosis nigricans {Choice A) is characterized by hyperpigmented, velvety patches seen on the neck, upper back, breasts, and axillae which is a marker of metabolic syndrome and diabetes. It is neither pre-malignant nor malignant and there are no atypical cells
on histopathologic examination.
Actinic keratosis (Choice B) is a premalignant lesion that may progress to squamous cell carcinoma. Clinically, lesions typically appear as light pink, ill-defined macules with a gritty texture in areas of prolonged sun exposure, such as the face, ears, and dorsal
hands.
Compound nevus (Choice C) is a benign proliferation of melanocytes located in both the epidermis and dermis They are very common. Compound nevi should not display asymmetry, border irregularity, or multiple colors. Development of compound and other
benign nevi should cease in the fourth to fifth decade. New or changing nevi after this time are concerning for melanoma.
Seborrheic keratosis (Choice E) is a benign proliferation of the epidermis; ' esions exhibit a greasy, adherent appearance. While seborrheic keratoses are often brown, this is due to the keratin produced by the epidermis rather than melanin.
Educational Objective: Lentigo maligna is a subtype of malignant melanoma that most commonly manifests on the sun-exposed skin of elderly patients. It is often slow growing and typically confined fo the dermal-epidermal junction.
© © © © f*
Previous Next Score Report Lab Values Calculator Help Pause
Exam Section 1: Item 10 ol 50 National Board of Medical Examiners^
Comprehensive Basic Science Self-Assessment
Y 10. A physician wishes to determine the proportion of newborns delivered at a local hospital who had a diagnosis of congenital heart disease within the past year. Which of the following statistical measurements best describes these data?
A ) Attributable risk
B) Incidence
C ) Odds ratio
D) Prevalence
E) Relative risk
Correct Answer: B.
incidence is an important epidemiological measure that assesses the rate of occurrence of new disease in a population at-risk. Incidence is the number of new cases expressed as a percentage of the total population at risk over a specified period. In this case: the
incidence would be the number of newborns with congenital heart disease divided by the total number of newborns in the study sample within the past year. For example: if 20 newborns are diagnosed with congenital heart disease out of a total of 1000 newborns
delivered at the local hospital within the past year, the incidence would be 2%.
Incorrect Answers: A, C, D. and E.
Attributable risk (Choice A) is a representation of the amount of risk that is associated with a particular exposure. Formally, its definition is the incidence rate in the exposed group minus the incidence rate in the control group. For example, if over the course of a
year in a group of 100 smokers there are five myocardial infarctions (incidence rate of 5%), and in a group of 100 non-smokers there are two myocardial infarctions (incidence rate of 2%), the risk attributable to smoking would be 5% minus 2%, or 3%.
Odds ratio (Choice C) is a comparison of the odds of an outcome occurring in the exposed group with the odds of that outcome occurring iin a nonexposed control group. Using the example of smokers and non-smokers referenced in Choice A, the odds of
myocardial infarction in the smoking group is 5/95 = 0.053. The odds of myocardial infarction in the non-smoking group is 2/93 = 0.0204. The odds ratio would simply be the odds of myocardial infarction in the smoking group divided by the odds of myocardial
infarction in the non-smoking group ((5/95)/{2/93)) = 2.53.
Prevalence (Choice D) is calculated as the ratio of the number of people with a disease divided by the total number of at-risk persons in a population at a particular point in time. This is also known as point prevalence or disease frequency. For example, if a survey
was conducted of a population of 1000 persons and 100 of these individuals were identified as having heart disease, the point prevalence of heart disease would be 100/1000 = 0.10: or 10%.
Relative risk (Choice E} compares the risk of an outcome in one group with the risk of an outcome in another group and is often used in cohort studies. It is similar to an odds ratio and can be confused with an odds ratio. The odds ratio can be used to approximate
relative risk when the disease or outcome state is rare. An example calculation of relative risk is as follows. In the described population of smokers referenced in Choices A and C, the risk of myocardial infarction in the non-smoking group is 2/100 = 0.020, or 2%.
The risk of myocardial infarction in the smoking group is 5/100 = 0.05, or 5%. The relative risk is, therefore, ((5/100) /(2/100)) = 2.50, meaning that the risk of myocardial infarction in the smoking group is 2.5 times the risk in the non-smoking group. Since, in this
example, myocardial Infarction is rare, the small disease assumption is valid, and the odds ratio approximates the relative risk (2.58-2.50).
Educational Objective: Incidence is a measure of the rate of occurrence of new disease in a population at-risk., which is distinct from prevalence, the measure of the current amount of disease burden in a population. Odds ratio is distinct from relative risk, although if
disease burden is rare, the odds ratio may be used to approximate relative risk.
© © ©
Previous Next Score Report Lab Values Calculator Help Pause
Exam Section t: Item 11 of 50 National Board of Medical Examiners^
Comprehensive Basic Science Self-Assessment
Y 11 A 25-year-old woman comes to the physician after her blood pressure was found to be 130/105 mm Hg at a health fair. She takes no medications. There is no family history of hypertension. Her last menstrual period was 1 week ago. Her blood pressure
today is 130/110 mm Hg. Bilateral abdominal bruits are heard Treatment with an angiotensin-converting enzyme (ACE) inhibitor will most likely have which of the following acute effects on this patient's renal function?
A ) Decreased concentrating ability secondary to renal angioedema
B) Decreased glomerular filtration rate secondary to dilation of efferent arterioles
C ) Decreased renal blood flow secondary to dilation of afferent arterioles
D } increased concentrating ability secondary to a change in permeability of the collecting duct
E) Interstitial nephritis secondary to allergic drug reaction
Correct Answer: B.
Fibromusctilar dysplasia of the renal artery is the most common cause of renal artery stenosis in younger and middle-aged women. Fibromuscular dysplasia is a non-inflammatory and non-atheroscierotrc angiopathy of medium-sized arteries (eg, renal, carotid) that
results in multifocal fibrous and muscular thickening of the arterial wali. which can lead to stenosis. Renal artery stenosis is a cause of secondary hypertension due to abnormal stimulation of the juxtaglomerular apparatus from low afferent blood flow leading to
excessive production of renin and angiotensin. The reduced afferent blood flow can result in progressive renal atrophy. Secondary hypertension should be considered in new-onset or treatment-resistant hypertension, or in younger, otherwise healthy patients. ACE
inhibitors are a first-line treatment for hypertension as they block the conversion of angiotensin  to angiotensin SI, which has direct vasoconstrictive effects as well as promotes salt and water retention. ACE inhibitors may result in a transient acute decrease in
glomerular filtration rate (GFR) secondary to dilation of efferent arterioles. This effect is more pronounced in patients with renal artery stenosis, as the baseline reduced afferent blood flow leaves the nephron dependent on efferent arteriole vasoconstriction
(mediated by angiotensin 11) to maintain adequate filtration pressure across the glomerulus. ACE inhibitors block this effect.
Incorrect Answers: A. C, D, and E.
Decreased concentrating ability secondary to renal angioedema (Choice A) is incorrect. ACE inhibitors are associated with angioedema as an adverse effect due to increased bradykinin levels, which may result in swelling of the face, lips, tongue, upper airway, and
gastrointestinal tract.
Decreased renal blood flow secondary to dilation of afferent arterioles (Choice C) is incorrect as dilation of the afferent arterioles would result in increased renal blood flow.
Increased concentrating ability secondary to a change in permeability of the collecting duct (Choice D) occurs with anti-diuretic hormone (ADH) re:ease from the pituitary or exogenous administration of ADH analogs. ADH results In increased aquaporin expression
on the luminal surface of collecting duct cells which increases the membrane permeability to water.
Interstitial nephritis secondary to allergic drug reaction (Choice E) is a possibility with many medications, but is commonly associated with sulfa-based diuretics, nonsteroidal anti-inflammatory medications, antibiotics, proton pump inhibitors, and rifampin. Patients
classically present with fever, hematuria, eosinophiluria, rash, and arthralgias.
Educational Objective: ACE inhibitors should be used with caution in patients with renal artery stenosis, as reduced efferent arteriole vasoconstriction may result in a decreased GFR.
© © © o o
Previous Next Score Report Lab Values Calculator Help Pause
Exam Section 1: Item 12 ol 50 National Board of Medical Examiners^
Comprehensive Basic Science Self-Assessment
Y 12. A 16-year-old boy is brought to the physician because of a 3-month history of shortness of breath while playing sports. He has no shortness of breath at rest. He says, ''Whenever I run around i cough, so I don't want to be on the basketball team anymore. "
He takes no medications and has no known allergies. There is a family history of hypertension and asthma. He is 165 cm (5 ft 5 in) taf! and weighs 63 kg (150 lb); BMI is 25 kg/m2 His respirations are l2/min; and blood pressure is 115/75 mm Hg. Cardiac
examination shows no abnormalities except for a midsystolic click at the apex. The lungs are clear to auscultation of the chest. Which of the following best explains this patient's symptoms?
A ) Deconditioning
B) Exercise-induced asthma
C ) Malingering
D ) Mitral valve prolapse
E) Thyroid disease
Correct Answer: B.
Exercise-induced asthma most likely accounts for this patients exertional dyspnea. Asthma is characterized by reversible obstruction of the bronchi secondary to hyperreactivity and airway inflammation. Patients present with episodes of wheezing: dry cough, and
dyspnea occurring during or shortly after exercise, relieved after rest or the use of bronchodilators. Physical examination during an exacerbation often reveals tachycardia, tachypnea, diffuse wheezes (or rhonchi); and prolonged expiration relative to inspiration.
Decreased tactile fremitus may be noted due to air trapping which decreases lung density (leading to reduced transmission of vibrations through the lung parenchyma to the body wall). Treatment is usually with a short acting bronchodilator (SABA) immediately
before exercise: although in patients with concomitant asthma not related to exercise: treatment is directed by the severity of underlying asthma, if patients do not tolerate SABAs, montelukast is an alternative option.
Incorrect Answers: A. C, D, and E.
Deconditioning (Choice A) could cause dyspnea, or the subjective experience of shortness of breath, but it should not cause a cough. It would also be unlikely in a young person with a normal BMI who has previously participated in sports without difficulty.
Malingering (Choice C) is defined by falsification of symptoms to obtain a secondary gain. This patient's symptoms have another possible explanation, exercise-induced asthma, and there is no clear secondary gain that he might obtain.
Mitral valve prolapse (MVP) (Choice D) is less likely to explain this patient's symptoms than exercise-induced asthma. The mid-systolic click of MVP is often, but not always: followed by a systolic murmur of mitral regurgitation (MR) when symptomatic and causing
cardiogenic pulmonary edema. Patients with congenital MVP often have physical findings including scoliosis, pectus excavatum: and low BMI. Symptoms of MVP (if sympfomatic) are more likely to include chest pain. palpitations: and lightheadedness in addition to
dyspnea.
Thyroid disease (Choice E) is unlikely in this patient, as other associated findings should be present. Dyspnea can result from hyperthyroidism or thyroid storm, but these conditions generally present with diaphoresis, weight loss, exophthalmos, and tremor. Thyroid
disease is also less common in male patients of this age group.
Educational Objective: Exercise-induced asthma presents with dyspnea, cough: and/or wheezing that begins during or shortly after exercise. It is usually treated with SABAs although montelukast is an alternative therapy.
Previous Next Score Report Lab Values Calculator Help Pause
Exam Section 1: Item 13 of 50 National Board of Medical Examiners^
Comprehensive Basic Science Self-Assessment
Y 13. A 25-year-old man comes to the physician because of a 3-day history of pain and swelling of his right leg. He has no history of major medical illness or recent trauma. Examination of the right lower extremity shows edema and tenderness. Duplex
uftrasonography of the right lower extremity shows a thrombus extending into the superficial femoral vein. Further studies show protein C deficiency. Inactivation of which of the following coagulation factors is most likely as a result of this deficiency in this
patient?
A ) Factors V (proaccelerin) and VIII (antihemophilic factor}
B) Factors V (proaccelerin) and IX (plasma thromboplastin component)
C ) Factors V (proaccelerin) and XI (plasma thromboplastin antecedent)
D ) Factors VIII (antihemophilic factor) and IX (plasma thromboplastin component)
E) Factors VIIII (antihemophilic factor) and XI (plasma thromboplastin antecedent)
F) Factors IX (plasma thromboplastin component) and XI (plasma thromboplastin antecedent)
Correct Ans1
wer: A.
Protein C inactivates factors V and VIII; deficiency of protein C leads to a hypercoagulable state predisposing to the development of both venous and arterial thrombi. The coagulation cascade consists of the intrinsic extrinsic, and common pathways. The intrinsic
pathway consists of sequential activation of factors XIl: XI. IV, and VIII, and activity of this pathway is measured by the partial thromboplastin time. The extrinsic pathway involves the activation of factor VII and is measured by the prothrombin time or the INFL Both
the intrinsic and extrinsic pathways can trigger the common pathway via activation of factor X to Xa. Activated factor X (factor Xa) has several functions, but one is to modify factor V and allow it to form a prothrombinase compfex with factor Xa. This complex helps
to form a fibrin clot. Factor Xa is also necessary for the conversion of prothrombin to thrombin. Thrombin binds to thrombomodulin on the surface of endothelial cells inducing a conformational change that allows it to activate protein C: while protein C localizes to the
endothelium by binding to the endothelial protein C receptor (EPCR). Activated protein C binds to the surface of activated platelets and degrades factors Va. and Villa, which is the means by which it exerts its anticoagulant effects.
Incorrect Answers: B. C, D: E; and F.
Factors V and IX (Choice B) and Factors V and XI (Choice C) are incorrect as factors IX and XI are not under inhibitory control by protein C. Activation of factor IX by Xla is an important step in the coagulation cascade and deficiency of factor IX is associated with
hemophilia. FactorXI activates factor IX in the intrinsic pathway.
Factors VIII and IX (Choice D). when activated, both take part in the intrinsic clotting cascade. While factor VI11 is under inhibitory control by protein C, factor IX is not.
Factors VIII and XI (Choice E) are incorrect as factor VIII is under inhibitory control by protein C but factor XI is not.
Factors IX and XI (Choice F) are both activated irr the intrinsic dotting cascade and serve to accelerate clotting. They are not influenced directly by protein C.
Educational Objective: Protein C is a natural anticoagulant that is activated by thrombin after thrombin binds to the endothelial surface. Activated protein C binds to the surfaces of activated platelets and degrades factors Va and Villa, thereby exerting negative
feedback on the clotting cascade.
© © © A o o
Previous Next Score Report Lab Values Calculator Help Pause
Exam Section 1: Item 14 ol 50 National Board of Medical Examiners^
Comprehensive Basic Science Self-Assessment
Y 14. Which of the following is most directly responsible for concentrating testosterone in the lumen of the seminiferous tubules?
A ) Androgen-binding protein
B) Follicle-stimulating hormone (FSH)
C ) FSH/gonadotropin-releasing hormone
D) InhJbrn
E) Luteinizing hormone
Correct Answer: A.
Androgen-binding protein (ABP) is produced by the Sertoli cells of the seminiferous tubules via the regulation of follicle-stimulating hormone (FSH). Testosterone is produced by Leydig ceils in the interstitium adjacent to the seminiferous tubules; the production of
which is regulated by of luteinizing lormone (LH). Once released into the lumen of the seminiferous tubuies; ABP facilitates spermatogenesis by binding to testosterone, allowing this otherwise lipophilic hormone to concentrate in the lumen. Normal
spermatogenesis requires high local concentrations of luminal testosterone.
Incorrect Answers: B, C, D; and E.
Follicle-stimulating hormone (FSH) (Choice B) is produced by gonadotropic cells in the anterior pituitary. It plays an important role in spermatogenesis by stimulating Sertoli cells to produce ABP and by directly stimulating sperm development. Its role in the
concentration of luminal testosterone lis indirect and mediated by ABP
FSH/gonadotropin-releasing hormone (Choice C) is produced by the hypothalamus and stimulates the production and release of FSH and LH from gonadotropic cells in the anterior pituitary. Its role in the concentration of testosterone in the lumen of the
seminiferous tubuies is indirect and is mediated by both FSH and ABP.
Inhibin (Choice D) is produced by Sertoli cells and exerts negative feedback on gonadotropic cells in the anterior pituitary to regulate the production of FSH.
Luteinizing hormone (Choice E) is also produced by gonadotropic cells in the anterior pituitary. It is important for stimu ating the production of testosterone but is not involved in the process of concentrating testosterone in the lumen of the seminiferous tubules.
Educational Objective: Androgen-binding protein (ABP) is produced by the Sertoli cells of the seminiferous tubules under the direction of foilic:
e-stimulating hormone. Once released into the lumen of the seminiferous tubules, ABP facilitates spermatogenesis by
binding to testosterone and concentrating it in the lumen of the tubules.
0 0 #
Previous Next Score Report Lab Values Calculator Help Pause
Exam Section 1: Item 15 ol 50 National Board of Medical Examiners^
Comprehensive Basic Science Self-Assessment
15. A 23-year-old woman has had the lesions shown in her mouth for 3 days. She has had frequent similar episodes over the past 15 years. The lesions are exacerbated by spicy, salty, and acidic food and drinks. They last approximately 1 week and resolve
spontaneously. Visits to the dentist seem to trigger the development of the sores. Which of the following is the most likely diagnosis?
A) Aphthous ulcers
6) Candidiasis
C) Geographic tongue
D ) Koplik spots
E) Leukoplakia
F) Lichen planus
G) Psoriasis
Correct Answer: A.
Aphthous ulcers are painful, round to oval, shallow oral ulcers. Tney are the most common cause of mouth sores and can be idiopathic or related to underlying conditions such as lupus erythematosus (in which case they are not painful) or Behget syndrome. They
may demonstrate pathergy, which is the development of new erosions at the site of a trauma, such as after a dental procedure. When these lesions develop recurrently a diagnosis of recurrent aphthous stomatitis is made. It is commonly seen in adolescence and
young adulthood, and episodes typically decrease with increasing age. The etiology is multifactoriat: but the lesions can be exacerbated by spicy acidic, or salty foods, as in this case. Stress can also lead to exacerbations. Recurrent aphthous stomatitis may be
treated by optimizing oral hygiene, avoiding exacerbating factors, and treating pain with topical anesthetics and coating agents.
ncorrect Answers: B. C. D. E. F. and G.
Oral candidiasis (Choice B) demonstrates thick, white plaques on the tongue or buccal mucosa, which can be scraped off with a tongue blade. It is commonly seen in immunosuppressed individuals, such as those with poorly controlled HIV infection, or patients
using a steroid inhaler and altering the normal oral microbiome.
Geographic tongue (Choice C) is a feature of psoriasis and often seen in chi dren. The tongue demonstrates a maze-like pattern of white, linear patches. Ulcers are not a typical feature.
Koplik spots (Choice D) are bright red macules with a bluish-white center on the buccal mucosa, which are a sign of an active measles infection. Koplik spots are accompanied by a prodromal fever, cough, coryza, conjunctivitis, and a confluent maculopapular
rash that starts at the headfneck and spreads to the trunk, excluding the palms and soles.
Leukoplakia (Choice E) refers to the development of white plaques in the mouth, which cannot be scraped off by a tongue blade, and are typically seen on the tongue or buccal mucosa. It may be due to an underlying Epstein-Barr virus infection and is common in
patients with HIV infection or malignancy.
Oral lichen planus (Choice F) is characterized by white patches with a stellate appearance on the buccal and gingival mucosa. Erosions can also occur, but the white, stellate patches will also be present, unlike in this case.
Psoriasis (Choice G) may affect the oral mucosa in the form of geographic tongue. However, it is more classically characterized by thick, salmon-colored plaques with si:very-white scale on the extensor extremities.
Educational Objective: Aphthous ulcers are painful, round to oval, shallow oral ulcers. They are the most common cause of mouth sores and can be exacerbated by certain foods, trauma, or emotiona stress.
m ft" in
Previous Next Scone Report Lab Values Calculator Help Pause
Exam Section 1: Item 16 ol 50 National Board of Medical Examiners^
Comprehensive Basic Science Self-Assessment
y
16. A 20-year-old woman comes to the emergency department 30 minutes after slipping on ice and extending her hand to break her fall. Palpation of the anatomic snuff-box produces pain. A wrist x-ray is most likely to show a fracture of which of the following
carpal bones?
A) Scaphoid
B) Lunate
C ) Triquetrum
D ) Pisiform
E) Trapezium
F) Trapezoid
G) Capitate
H ) Hamate
Correct Answer: A.
Falling onto an outstretched hand can iead to traumatic injuries such as distal radius fractures, elbow dislocations, or fractures of the carpal bones of the wrist. A common pattern of injury with this mechanism is a scaphoid fracture. The scaphoid bone is part of
the lateral column of the wrist and supports force transmission from the hand to the lateral aspect of the radius. Fractures of this bone typically present with lateral wrist pain and tenderness in the anatomic snuff-box, which is the dorsal depression between
extensor poMicis iongus and abductor pollicis longus. The scaphoid has a blood supply that proceeds from distal to proximal. Displacement of a fracture of this bone may lead to decreased blood supply of the proximal fragment, leading to avascular necrosis and
debilitating wrist pain and deformity. Because of this, it is important to identify and appropriately treat scaphoid fractures.
Incorrect Answers: B. C, D, E. F, G, and H.
Fractures of the lunate (Choice B) are an uncommon injury. More commonly with high energy injuries to the wrist, the capitate and the remaining carpal bones may dislocate from the concave surface of the lunate. This is known as a perilunate dislocation and
may lead to acute compression of the median nerve.
The triquetrum (Choice C) is a carpal bone located in the ulnar aspect of the wrist. Fractures in this location are uncommon. If fractured, it would present with tenderness along the ulnar aspect of the wrist.
The pisiform (Choice D) is a carpal bone in the ulnar aspect of the wrist. It is a pea-shaped sesamoid bone that can be mistaken for a fracture fragment. Fractures of the pisiform are uncommon.
The trapezium (Choice E) is the bone at the base of the thumb that can also be palpated in the floor of the anatomic snuff-box. Fracture of the trapezium is uncommon and typically presents with pain after trauma to the thumb.
The trapezoid (Choice F) is a wedge-shaped carpal bone just proximal to the second metacarpal. Fracture of the trapezoid is an uncommon carpal injury.
The capitate (Choice G) is a larger carpal bone in the center of the wrist. It articulates with the lunate and is dislocated in a perilunate dislocation.
The hamate (Choice H) is a carpal bone in the ulnar aspect of the wrist that has a process along its volar surface, referred to as the hook of the hamate, that ads as a covering for the finger flexor tendons of the uJnar digits. Fracture of the hook of the hamate can
occur while placing a high load through the wrist while holding a handle (eg, baseball bat, golf club, sledgehammer). This injury can lead to compression of the ulnar nerve in Guyon canal and irritation or impingement of the adjacent tendons.
Educational Objective: Fracture of the scaphoid presents with radial wrist pain and tenderness in the anatomic snuff-box, typically following a fall on an outstretched hand. Identification and appropriate treatment of this injury is important as the retrograde blood
supply to this bone places it at risk for avascular necrosis of the proximal fragment.
O m m rts
Previous Next Score Report Lab Values Calculator Help Pause
Exam Section 1: Item 17 of 50 National Board of Medical Examiners^
Comprehensive Basic Science Self-Assessment
Y 17. A 76-year-old man undergoes laparotomy for resection of an abdominal aortic aneurysm. During the procedure, an incidental finding of acquired coionic diverticula is made. The diverticula in this patient are most likely present in which of the following?
A ) Ascending colon
B) Cecum
C ) Descending colon
D) Sigmoid colon
E) Transverse colon
Correct Answer: D.
The sigmoid colon is the most common location for diverticula to form. Diverticula are outpouchings of the mucosal and submucosal layers into the muscular layer that occur at weak points in the gut wall where the small arterioles of the vasa recta penetrate, it is
hypothesized that abnormal motility in the colon causes increased intraluminal pressure with subsequent herniation of the mucosa through weak points in the colonic wall. This may happen in the sigmoid coton because the diameter of the sigmoid is smaller than
other parts of the colon, so abnormal peristalsis in this area causes higher intraluminal pressure compared to other segments. Risk factors for development of diverticula include obesity and a diet high in red meat and low in fiber. Diverticulosis predisposes to lower
gastrointestinal bleeding (Gl). It also predisposes to diverticulitis, a bacterial infection of a diverticulum that leads to a local inflammatory response. Diverticulitis presents classically with fever, left lower quadrant abdominal pain, and occasionally with bloody diarrhea.
Incorrect Answers: A, B. C. and E.
The ascending colon (Choice A) can be a site of colorectal carcinoma (CRC). Because stool is generally liquid in the ascending colon and tumors in this location tend not to be exophytic C:RC can present [ate as symptoms are less common. While diverticula may
occur in the ascending colon, it is a less common location than the sigmoid. Similarly, the transverse colon (Choice E} and descending colon (Choice C) may develop diverticula, but these sites are less common than the sigmoid colon.
Cecum (Choice B) is the junction between the ileum and the ascending colon. It is not a common site of diverticuEosis, but since the appendix lies in close proximity to it, the cecum can occasionally become inflamed in severe, acute appendicitis.
Educational Objective: Diverficufosis refers to a condition in which the colonic mucosa and submucosa herniate into the muscular layer at weak points where the vasa recta penetrate the colonic wall. Risk factors include chronic constipation and a low fiber diet. The
sigmoid colon is the most frequently involved part of the colon as its smaller diameter predisposes to greater intraluminal pressure during peristalsis, thereby increasing the risk for diverticula formation.
* It
Previous Next Score Report Lab Values Calculator Help Pause
Exam Section 1: Item 13 ol 50 National Board of Medical Examiners^
Comprehensive Basic Science Self-Assessment
Y 18. During a period of 36 hours, an 80-year-old woman has increasingly severe abdominal pain followed by fever chills, tachycardia, hypotension and. finally, shock. Blood cultures grow Escherichia coii Her condition worsens and. despite supportive therapy
and antibiotics; she dies 4 days after the onset of the illness. Which of the following is the most likely cause of the initial hypotension?
A) Excessive production of nitric oxide
B) Generation of hydrogen peroxide
G) Hemorrhage
D } Induction of endothelial adhesion molecuEes
E) Platelet aggregation
Correct Answer: A.
Sepsis is a systemic inflammatory syndrome that results from a dysregulated and exaggerated immune response to an infection. Sepsis can be complicated by shock and multiorgan failure with a high mortality rate. Septic shock is characterized by an impaired
response of the vasculature to vasoconstricting stimuli with markedly decreased systemic vascular resistance, tachycardia: increased cardiac output, oliguria; and lactic acidosis. Excessive production of nitric oxide is associated with hypotension in the setting of
sepsis. Inducible nitric oxide synthetase is a nitric oxide-producing enzyme that is upregulated through tyrosine kinase activation in response to proinflammatory cytokines and binding by Hpopolysaccharides. Nitric oxide activates guanyiate cyclase in vascular
smooth muscle resulting in muscle relaxation and vasodilation because of increased intracellular cyclic guanosine monophosphate concentration.
Incorrect Answers: B, C: D: and E.
Generation of hydrogen peroxide (Choice B) occurs in phagolysosomes by the enzyme superoxide dismutase: which utilizes free oxygen radicals produced by NADPH oxidase. Excess free radical production is associated with host tissue injury.
Hemorrhage (Choice C) may result in shock secondary to hypovolemia and decreased oxygen carrying capacity of the blood secondary to loss of hemoglobin. This patient's abdominal pain is likely the result of an infectious enteritis, colitis, or peritonitis, and is less
likely from a ruptured abdominal aortic aneurysm, which may cause shock from internal exsanguination.
Induction of endothelial adhesion molecules (Choice D) is a key step in the recruitment and migration of leukocytes to sites of infection and injury. The action of nitric oxide on the vascular smooth muscle results in hypotension, whereas the expression of endothelial
adhesion molecuEes is involved in the immune response to localized infection.
Platelet aggregation (Choice E) and activation occurs in response to inflammation and helps promote the innate immune response, although it does not directly cause the patient's hypotension. Diffuse platelet aggregation may result in thrombocytopenia in the
setting of sepsis.
Educational Objective: Sepsis is a dysregulated systemic inflammatory syndrome that may occur in response to infection. It may progress to hypotension and septic shock. Excessive nitric oxide production causing diffuse vasodilation is one of the mechanisms of
hypotension in sepsis.
O 0 © 0 0
Previous Next Score Report Lab Values Calculator Help Pause
Exam Section 1: Item 19 of 50 National Board of Medical Examiners^
Comprehensive Basic Science Self-Assessment
Y 19. A G5-year-old woman comes to the physician because of a 3-month history of headache, weakness of her arms, and left flank pain: she also has had a 14-kg (31-lb) weight loss during this period. Physical examination shows weakness of the proximal upper
and lower extremity muscles. There is augmentation of strength with repetitive testing of the deltoid muscles. An MRI of the brain shows a single well-demarcated mass surrounded by edema in the right frontal lobe. A stereotactic biopsy specimen of the
:esion shows a malignant small blue cell neoplasm that expresses cytokeratin, chromogranirv and synaptophysin. Which of the following is the most likely diagnosis?
A ) Anaplastic ependymoma
B) Extranodal primary central nervous system lymphoma
C ) Glioblastoma multiforme
D ) Primary cerebral neuroblastoma
E) Pulmonary smalt cel: carcinoma metastatic to the brain
Correct Answer: E
The patient's presentation is most consistent with pulmonary smali cell carcinoma metastatic to the brain. Pulmonary small cell tumors are typically centrally located in the lungs and associated with tobacco use. They are neoplasms of neuroendocrine ceils and may
be associated with numerous paraneoplastic syndromes, inctuding Cushing syndrome due to adrenocorticotropic hormone production, syndrome of inappropriate antidiuretic hormone, Lambert-Eaton myasthenic syndrome due to presynaptic calcium channel
antibody production, and paraneoplastic myelitis, encephalitis, and subacute cerebellar degeneration. Proximal extremity weakness and augmentation of strength with repetitive testing of the deltoid muscles are suggestive of Lambert-Eaton myasthenic syndrome.
Histologic features of pulmonary small cell carcinoma include small dark blue tumor cells lacking nucleoli with a high nuclear to cytoplasm ratio. The brain is a common site for metastatic disease.
Incorrect Answers: A. B, C. and 0.
Anaplastic ependymoma (Choice A) is a central nervous system neoplasm formed from ependymal cells. Location typically involves the fourth ventricle. Histologic characteristics include perivascular pseudorosettes formed by malignant cells arranged around a
blood vessel.
Extranodal primary central nervous system lymphoma (Choice B) is a rare type of malignant non-Hadgkin lymphoma, often associated with an underlying immunodeficiency syndrome (eg, AIDS). Histologic examination may reveal large, atypical lymphocytes.
Glioblastoma multiforme (Choice C) is a malignant primary brain tumor characterized by central necrosis on histology. Imaging features include an expansile mass crossing the corpus callosum with surrounding vasogenic edema. Seizures are a common presenting
symptom, as are headaches and focal neurologic deficits.
Primary cerebral neuroblastoma (Choice D) is a malignancy of neuroendocrine cells associated with sympathetic nervous tissue. Neuroblastomas may arise from the adrenal glands; the sympathetic chain, or the central nervous system, and may secrete
catecholamines. Metastatic pulmonary small cell carcinoma is more common in adults than a primary cerebral neuroblastoma.
Educational Objective: Solitary brain lesions may be secondary to a primary central nervous system malignancy: metastatic disease, infection or abscess. Histologic analysis and clinical features can help narrow the diagnosis. Pulmonary small cell carcinomas are
neuroendocrine tumors associated with paraneoplastic syndromes and brain metastases.
© © © © f*
Previous Next Score Report Lab Values Calculator Help Pause
Exam Section 1: Item 20 ol 50 National Board of Medical Examiners^
Comprehensive Basic Science Self-Assessment
Y 20. Failure of normal differentiation of the endoderm in the embryonic lung bud is most likely to affect the development of which of the following?
A ) Capillary patterns
B) Cartilage in bronchi
C) Smooth muscle on the bronchi
D) Surfactant secretion
E) Tracheal rings
Correct Answer: D.
Endoderm is one of the three primary embryonic germ layers and composes the innermost layer of the early developing organism. Endoderm derivatives include the epithelial linings of the respiratory tract, gastrointestinal tract biliary system, genitourinary tract,
vagina, and middle ear. Organs that arise from the endoderm include the liver, parathyroid glands,, thymus, pancreas, and the follicular and parafollicular cells of the thyroid. Surfactant secretion in the mature lung is achieved by type II pneumocytes, a component of
the respiratory epithelium. Defective differentiation of the endoderm in the embryonic lung bud would most ikely result in impaired development of type II pneumocytes and reduced secretion of surfactant.
Incorrect Answers: A, B. C, and E.
Capillary patterns (Choice A), cartilage in bronchi (Choice B), smooth muscle on the bronchi (Choice C), and tracheal rings (Choice b} are all derivatives of the mesoderm. Mesoderm is the middle embryonic germ layer and primarily responsible for development of
connective tissue structures, including muscle, dermis, bone, cartilage, dura mater, the cardiovascular system, lymphatic system, blood components, kidneys, adrenal cortex, and reproductive organs.
Educationa Objective: Ceils of the respiratory epithelium arise from the embryonic endoderm germ layer. This includes type I pneumocytes. which form the simple squamous epithelium of the alveoli, and type II pneumocytes, which secrete surfactant.
0 0 & % 0*
Previous Next Score Report Lab Values Calculator Help Pause
Exam Section 1: Item 21 ol 50 National Board of Medical Examiners^
Comprehensive Basic Science Self-Assessment
Y 21. A G7-year-old woman comes to the physician for a health maintenance examination. Her brother and mother have a history of colon cancer. The physician recommends colonoscopy: taut the patient says that she would prefer only for her stool to be tested
for blood. The physician explains that testing the stool for occult blood is not appropriate in this case. The physician is most likely concerned about which of the following regarding this test?
A) Low sensitivity
B) Low specificity
C ) Potential for a false-positive result
D } Uncertain negative predictive value
E) Uncertain positive predictive value
Correct Answer: A.
Sensitivity is the ability of a test to detect a disease if it is present. A test is described as sensitive if it has a high likelihood of disease detection, and therefore a low likelihood of false negativity. High sensitivity is therefore useful in ruling out a disease. This is
because a negative result from a high sensitivity test indicates a low likelihood that the disease is present. Because of this, high sensitivity tests are useful for screening in which proving the absence of disease and limiting the number of false negative results are of
utmost importance. In the case of cancer screening, a highly sensitive test allows the clinician to be confident that a negative test means that the patient is disease-free. In contrast, a test with poor sensitivity, if negative, does not provide strong evidence or
confidence that the patient does not have the disease. In the case described, the fecal occult blood test demonstrates both a low sensitivity and specificity and is of limited use to the physician and the patient for ruling out cancer.
incorrect Answers: B, C: D: and E.
Low specificity (Choice B) describes a test that is subject to false-positive errors, meaning that a positive test does not have a high likelihood of disease. High specificity is required to confirm a diagnosis. An example of high specificity testing for colon cancer would
be a colonoscopy with biopsy of a lesion with staining and molecular testing under microscopy. Direct visualization of malignant cells on microscopy is highly specific for a cancer diagnosis, that is, there is a low likelihood of a false cancer diagnosis.
Potential for a false-positive result (Choice C) is possible with a fecal occult blood test. For example, a bleeding hemorrhoid or taking an iron supplement can cause a positive fecal occult blood test. This is unlikely, however, to cause concern by the physician in this
scenario as a false positive test does not necessarily put the patient at risk for an undiagnosed colorectal cancer, although it may lead to unnecessary further diagnostic examinations. Missing an early cancer diagnosis due to poor test sensitivity could lead to early
mortality and is a more pressing issue.
Uncertain negative predictive value (Choice D) is not of immediate concern in this scenario. Negative predictive value is based on both the test sensitivity and the pretest probability of the patient having the disease. Although certain individuals have higher or lower
risk of colon cancer based on lifestyle and genetics, pretest probability and alterations in the negative predictive value do not alter the need for a sensitive test for screening purposes.
Uncertain positive predictive value (Choice E) is also not an immediate concern. Positive predictive value is based on both test specificity and the pretest probability of disease. Positive predictive value is the likelihood that a person has a disease, given a positive
test. A positive predictive value is of greater importance in confirmatory testing.
Educational Objective: High sensitivity tests are required for effective disease screening. High specificity tests are necessary for confirmation of the disease. Negative and positive predictive values are functions not only of sensitivity and specificity, but also of the
pretest probability of the disease.
O 0 ©
Previous Next Score Report Lab Values Calculator Help Pause
Exam Section 1: Item 22 ol 50 National Board of Medical Examiners^
Comprehensive Basic Science Self-Assessment
Y 22. A 5-year-old boy is brought to the emergency department after ingesting 10 oz of a household cleaning solvent. He is treated for acute hepatic and renal failure for 1 week and then discharged. During the next month; regeneration of this boy's mature
hepatocytes and renal tubular epithelial cells will be accomplished mostly by which of the following mechanisms?
A ) Activation of stem cells to enter G1 phase of the cell cycle
B) Decreased apoptosis at GrMtransition of the cell cycle
C ) Recruitment of cells from G0 into the cell cycle
D } Shortened time for progression of cells through the cell cycle
E) Terminal differentiation by cells exiting from the cell cycle
Correct Answer: C.
When hepatocytes or renal tubular epithelial cells are destroyed, the remaining cells are recruited from quiescence (the phase) to re-enter the cell cycle. In healthy patients, the majority of hepatocytes and renal tubular epithelial cells are in the GQ phase. When
cells are destroyed such as in acute liver or renal failure, genes are induced that prime remaining ceils to re-enter the cell cycle from quiescence. These cells transition from GQ phase to G1 phase, where growth factors engender cell growth. Ceils that grow
sufficiently surpass the restriction point, at which point they are committed to DNA replication and cell division via mitosis. After the G-j restriction point, the cells transition to the S phase: when DNA replicates. The cells enter another growth phase: the G2 phase,
and finally the M phase, when mitosis occurs and hepatocytes and renal tubular epithelial cells regenerate.
Incorrect Answers: A. Bf D. and E.
Activation of stem cells to enter the G1 phase of the cell cycle (Choice A) does not play a major role in the regeneration of hepatocytes or renal tubular epithelial cells. Stem cells from the bone marrow or within the liver/kidney itself may minorly contribute to
regeneration, but the recruitment of the large population of quiescent cells into the cell cycle is more crucial.
Decreased apoptosis at G
^
-M transition of the cell cycle (Choice B) and shortened time for progression of cells through the cell cycle (Choice D) would not lead to regeneration of hepatocytes or renal tubular epithelial cells. Most cells are quiescent (not in the cell
cycle) at baseline so decreasing apoptosis or shortening the cell cycle time would not lead to an appreciable increase in cells.
Terminal differentiation by cells exiting from the cell cycle (Choice E) does not occur after hepatic or renal damage and would prevent cells' future ability to regenerate. Many cell types (eg, skeletal muscle cells) terminally differentiate and lose their ability to
regenerate.
Educational Objective: The vast majority of hepatocytes and renal tubular epithelial cells are in the GQ phase (quiescence) at baseline. When cells are destroyed, the remaining cells re-enter the cell cycle at the G1 phase to grow and divide, leading to regeneration.
0 0 ^
Previous Next Score Report Lab Values Calculator Help Pause
Exam Section 1: Item 23 of 50 National Board of Medical Examiners^
Comprehensive Basic Science Self-Assessment
Y 23. A 17-year-old boy is brought to the physician by his mother because she is concerned that his puberty is delayed. The mother states: "He is so short. His father is 6 feet 5 inches tall. I don't understand why he has not had his growth spurt." When the mother
leaves the room, the patient states:
"I'm fine. ! don't know what's the matter with her. She wants me to be tall like my dad." The patient is 175 cm (5 ft 9 in) tall and weighs 70 kg (155 lb); BMI is 23 kg/m2 Sexual development is Tanner stage 4. in addition to
reassuring the mother that her son is fine, which of the following is the most appropriate initial statement by the physician to the mother?
A ) "Since your son is fine with his height you should try to accept him as he is. "
B) "Tel: me more about your concerns about your son's height.
"
C ) "We’ll do some blood tests just to be sure that all your son's hormone levels are okay."
D ) "Your son is average for his height and weight."
E) "Your son is not going to be any taller."
Correct Answer: B.
When patients or patients' family members express medical concems: physicians should initially ask open-ended questions to explore the understanding and fears of the patient or family member. The physician can then tailor further discussion and reassurances to
address these knowledge gaps and fears. Asking open-ended questions also invites the family member to elaborate on their concerns, as there may be medically or psychfatrically relevant details that the family member reveals on further discussion. Further,
listening to the specific concerns of the patient or family members will improve therapeutic alliance.
Incorrect Answers: A. C; D, and E.
Giving parental advice (Choice A) would be unwarranted in this situation and outside of the physician's scope of practice. If a parent's behavior is clearly affecting the menta or physical health of the patient, the physician may tactfully bring the issue to the parent's
attention. However this patient's health is not clearly impacted by his mother's concern.
Physicians should refrain from ordering tests that are medically unnecessary based on patient or family concern (Choice C). The physician should instead reassure and educate after listening to the patient or family's specific concerns.
Saying that the patient is average for his height and weight (Choice D) would not address this mother's concern about the patient being shorter than his father Tnis statement would also prevent elaboration of the mother's specific concerns, understanding, and
fears.
Informing the patient's mother that her son is not going to be any taller (Choice E) would not be accurate or reassuring. This statement would also prevent elaboration of the mother’s specific concerns, understanding, and fears.
Educational Objective: When patients or families express medical concerns physicians should ask open-ended questions to elucidate the specific nature of the concern and the patient's or family member's understanding. The physician can then tailor further
discussion to address knowledge gaps and specific fears, and may learn additional, medically relevant details about the concern.
© © ©
Previous Next Score Report Lab Values Calculator Help Pause
Exam Section 1: Item 24 ol 50 National Board of Medical Examiners^
Comprehensive Basic Science Self-Assessment
y
24. A 34-year-old woman is admitted to the hospital for treatment of pulmonary tuberculosis. Infliximab therapy was initiated G months ago for severe Crohn disease. This pharmacotherapy most likely inhibited which of the following immunologic functions in this
patient?
A ) Activation of nuclear factor KB to induce expression of interleukin-10 (IL-10)
B) Direct toxicity to the causal organism
C ) Maintenance of granulomas
D } Recruitment of segmented neutrophils to ingest and kill the bacteria
E) Stimulation of B lymphocytes to produce neutralizing antibodies against the causal organism
Correct Answer: C.
Tumor necrosis factor-a (TNF-a) is a cytokine secreted by macrophages, which supports granuloma formation. Granulomas are collections of histiocytes, or macrophages with abundant pink cytoplasm that often contain multi-nucleated giant cells and are
surrounded by lymphocytes One purpose of granuloma formation is to sequester an infection or foreign body. Monoclonal antibody therapy targeted against TNF-a increases a patients risk for active mycobacterial infection because it stops the production of TNF-a,
leading to breakdown of the granuloma and release of any contained organism. Monoclonal antibodies against TNF-a that are specifically used to treat Crohn disease include infliximab, adalimumab, and cerfclizumab. These medications are known to increase the
risk of reactivating latent Mycobacterium tuberculosis because of their deleterious effect on granuloma formation and maintenance. All patients who are considered for treatment with these agents must undergo screening for latent M tuberculosis infection. If
positive, treatment for latent M. tuberculosis with nine months of isoniazid is indicated.
Incorrect Answers: A. B, D. and E.
Activation of nuclear factor KB to induce expression of interleukin-10 (IL-10) {Choice A) occurs within the Th2 cell type. 1L-1G attenuates the immune response by inhibiting activated macrophages and decreasing expression of Th1 cytokines. Inhibition of this IL-1D
would therefore stimulate the immune process against infectious organisms, not decrease it.
TNF- a is a cytokine that maintains granulomas and assists in walling off infections such as tuberculosis. It does not have any direct toxicity to the causal organism (Choice B),and thus this is not the mechanism by which anti-TNF-a monoclonal antibodies increase
the risk of latent tuberculosis reactivation.
IL-8, not TNF-a, is responsible for the recruitment of segmented neutrophils to ingest and kill the bacteria (Choice D). This process is not affected by the inhibition of TNF-a.
Rituximab, an airti-CD20 monoclonal antibody, diminishes B lymphocytes and is used in chemotherapy regimens for lymphoma. It is also used in the treatment of autoimmune conditions including vasculitis, rheumatoid arthritis, and pemphigus vulgaris. This
monoclonal antibody, not a TNF-a inhibitor, prevents stimulation of B lymphocytes to produce neutralizing antibodies against the causal organism (Choice E).
Educational Objective: The use of monoclonal antibody therapy against TNF-a is associated with an increased risk of reactivated latent M. tuberculosis secondary to inadequate granuloma maintenance. Because of this, all prospective candidates for this therapy
should be screened for latent M. tuberculosis infection and accordingly treated if positive.
© © © © f*
Previous Next Score Report Lab Values Calculator Help Pause
Exam Section 1: Item 25 ol 50 National Board of Metlicai Examiners^
Comprehensive Basic Science Self-Assessment
y
25. An 8-year-old boy continues to bleed excessively after tooth extraction. Prothrombin time, bleeding time, and platelet count are within the reference range. Partial thromboplastin time is prolonged but corrects after addition to the assay chamber of plasma
from a patient with hemophilia A. Which of the following is the most likely diagnosis?
A ) Acute disseminated intravascular coagulation
B) Factor V (proaccelenn) deficiency
C ) Factor Vt! (proconvertin) deficiency
D ) Hemophilia A
E) Hemophilia B
F) Immune thrombocytopenic purpura
G) von Willebrand disease
Correct Answer: E
Hemophilia B is an X-linked bleeding disorder caused by absent, decreased, or dysfunctional factor IX and most likely explains this patient's prolonged bleeding and increased partial thromboplastin time (PIT). As patients with hemophilia A lack factor VIII but
have normal levels of factor IXf addition of plasma from a patient with hemophilia A would correct the PTT as demonstrated m this patient with hemophilia B. Factor IX is a component of the intrinsic clotting cascade and serves to activate factor X to Xa. which
subsequently converts prothrombin to thrombin and facilitates the formation of a fibrin clot. The activity of the coagulation factors in the intrinsic coagulation cascade is measured by the PTT while the activity of the extrinsic pathway is measured by the
prothrombin time (PT)f
which is normal in this patient. The clinical severity of hemophilia 6 is variable. Patients with severe disease present early in life with easy bruising, bleeding following a minor procedure, or hemarthrosis. Patients with less severe disease
may not present until they experience an event such as trauma or surgery Treatment includes replacement of the deficient factor.
Incorrect Answers: A. R, C. D, F, and G.
Acute disseminated intravascular coagulation (Choice A) is a syndrome characterized by overwhelming activation of the clotting cascade often precipitated by malignancy, sepsis, or obstetrical emergencies. Endothelial dysfunction leads to the formation of
microthrombi and depletion of coagulation factors. Microthrombi cause shearing stress on erythrocytes leading to microangiopathic hemolytic anemia, while the depletion of coagulation factors manifests as a prolonged FT and PTT and increases the risk of major
bleeding.
Factor V (proaccelerin) deficiency (Choice B) is a rare inherited bleeding disorder that may present with mucocutaneous bleeding or major bleeding following trauma or surgery. Factor V is required as a cofactor for the formation of thrombin in the common
pathway of the coagulation cascade. Deficiency is treated with fresh frozen plasma (FFP)f
which contains factor V.
Factor VII (proconvertin) deficiency (Choice C) Is a rare bleeding disorder with a spectrum of clinical severity Patients who are most affected present with heavy menstrual bleeding or bleeding following invasive procedures. Treatment is with factor replacement,
prothrombin complex concentrate, or FFP.
Hemophilia A (Choice D) is an X-linked bleeding disorder that presents similarly to hemophilia B and is caused by an absent or reduced level of factor VIII. Addition of plasma from another patient with hemophilia A would not correct the PTT.
Immune thrombocytopenic purpura (Choice F) is caused by circulating antibodies against platelets that leads to thrombocytopenia. The PT and PTT are normal Bleeding, if it occurs, tends to be mucocutaneous.
von Willebrand disease (Choice G) is one of the most common hereditary bleeding disorders and is due to quantitative or qualitative abnormality of von Wil ebrand factor, which binds platelets and subendothekai collagen in primary hemostasis. Impaired platelet
adherence leads to a prolonged bleeding time. It can present with epistaxis, gingival bleeding, petechiae, easy bruising, and menorrhagia.
Educational Objective: Hemophilia B is an X-linked bleeding disorder that is caused by a deficiency in factor IX leading to an increased PTT. Symptoms depend on severity but include prolonged bleeding following invasive procedures or trauma, easy bruising
and hemarthrosis. Treatment is with recombinant factor IX.
o m m
Previous Next Score Report Lab Values Calculator Help Pause
Exam Section 1: Item 26 ol 50 National Board of Metlicai Examiners^
Comprehensive Basic Science Self-Assessment
y
26. The diagram shows the major factors that determine blood pressure. Which of the fallowing labeled factors is affected most by an aradrenergtc antagonist?
^Centralnervous system
'
i
® Peripheral resistance
(arteriolar )
1=fCardiac output j x
Blood pressure
©
^Stroke volume ] fHeart rate
CD
"Contractility j £Venous return
"
^
Blood volume! Capacitance vessel tone
' (venolar)
©
A )
B)
C )
D)
El
Correct Answer: C.
The autonomic nervous system plays a primary role in the maintenance of blood pressure via its effects on peripheral arteriolar resistance: heart rate, myocardial contractility: venous capacitance, and to an indirect degree., blood volume. The sympathetic nervous
system primarily acts through a- and (3-adrenergic receptors, which are stimulated by dopamine, epinephrine: and norepinephrine. In relation to blood pressure regulation, the a t receptor leads to smooth muscle contraction especially of the vasculature, which
increases blood pressure by increasing systemic vascular resistance. The p;receptor in comparison, causes increases iin heart rate, myocardiat contractility, and renin release, which all lead to increases in blood pressure. An antagonist at the a1 receptor would
decrease vascular smooth muscle contraction to a greater extent around arteries, leading to vascular dilation and decreased blood pressure as mean arterial pressure is directly related to cardiac output and systemic vascular resistance.
Incorrect Answers: A. Ei. D, and E.
Central nervous system inputs to cardiac output (Choice A) can occur through the limbic system in response to strong emotions or anticipation of physical activity or through central components of the autonomic nervous system (eg. brainstem, spinal autonomic
preganglionic neurons). These autonomic stimuli to the heart would be less affected by an a1 receptor antagonist than peripheral resistance, as the stimuli would also include effects on the p,receptor By contrast, central a2-adrenoreceptors are the target of the
agonist clonidine, which results in diminished sympathetic tone,, forming the basis for the use of clonidine in hypertensive emergency.
Heart rate (Choice B) is primarily affected by the pi receptor, with agonism leading to increases in heart rate. As part of the parasympathetic nervous system, heart rate is decreased by M2 receptors. a1 receptor antagonists do not affect heart rate directly, though
can cause reflex tachycardia.
Blood volume (Choice D) is affected by changes in red blood cells and plasma volume, such that anemia or hemorrhage would decrease blood volume. However changes in plasma volume can also be mediated by diuretics (decreasing plasma volume) or renin
release (increasing plasma volume). a1 receptors do not moderate blood volume.
Capacitance vessel tone (Choice E} describes the ability of the vessel to hold a volume of blood at a specific blood pressure. The capacity of the venular compartment increases with decreased somatic muscle movement, valvular dysfunction, and nitroglycerin
administration. Venule tone decreases with ,receptor blockade, but to a lesser extent than periphera arteriolar resistance as there is minimal vascular smooth muscle in venules compared to arterioles.
Educational Objective: The autonomic nervous system plays a primary role in the maintenance of blood pressure via its effects on peripheral arteriolar resistance, heart rate, myocardial contractility, venous capacitance, and to an indirect degree, blood volume.
aradrenergic receptors primarily modulate mean arterial- pressure by increasing peripheral arteriolar resistance.
© © m -
"s
7
Previous Next Scone Report Lab Values Calculator Help Pause
Exam Section 1: Item 27 of 50 National Board of Metlicaf Examiners^
Comprehensive Basic Science Self-Assessment
y
-Y'
NT
VI
V
5 iVft
3!
V EZI
aVL V2 v?
3IJ PW VS
R
! /I
:
VI
I 1
v__._.
J
II
U. _l
'V . .
.l~A.
...
VS
27. A previously healthy 21-year-old woman comes to the office because of a 2-month history of shortness of breath and fatigue. Her most recent menstrual period was 3 months ago. Menses previously had occurred at regular 28-day intervals. She tells the
physician that she thinks she may be pregnant. She takes no medications and has not seen a physician for several years. She appears healthy. She is 160 cm (5 ft 3 in) tall and weighs 54 kg (120 lb); BMI is 21 kg/m2 Vital signs are within normal limits. The
lungs are dear Cardiac examination shows a normal S1; a widely split S2 that does not change with respiration, and a grade 3/6 holosystoMc murmur that is loudest at the lower left sternal border and radiates to the upper left sternal border EGG is shown.
The most likely cause of these findings is dysfunction of which of the following structures?
A) Atrial septum
B) Ductus arteriosus
C ) interventricular septum
D ) Pulmonic valve
E) Tricuspid valve
Correct Answer: A.
Atrial septal defect is a common congenital malformation of the interatrial septum The most common type is an ostium secundum defect although ostium primum defects are associated with trisomy 21. The atrial septal defect results in a left-to-right shunt with
abnormal flow of blood from the left atrium to the right atrium, resulting in relative volume overload of the right atrium and ventricle. This increased stroke volume of the right ventricle results in delayed closure of the pulmonic valve, which presents as a fixed, split
S2l
and low-grade physiologic ejection murmur on cardiac auscultation. The increased right heart volumes also result in a prominent right ventricular impulse on physical exam and may present an increased risk for the development of a right bundle branch block,
which may be present on ECG as seen in this case. If the atrial septal defect remains uncorrected, it can result in the development of Eisenmenger syndrome secondary to prolonged pulmonary vasculature remodeling resulting in pulmonary arterial hypertension
and shunt reversal leading to cyanosis. Asymptomatic atrial septal defects may become clinically significant in the setting of increased blood flow (such as during pregnancy).
Incorrect Answers: B, C: D. and E.
A patent ductus arteriosus (Choice B) is a persistent extracardiac conduit between the aorta and the pulmonary artery that has failed to obliterate after birth. It results in a continuous, machine-like murmur best heard in the left second intercostal space, radiating to
the clavicle.
Defects of the interventricular septum (Choice C) are characterized by a hoiosystolic murmur best heard in the left lower sternal border They do not classically result in a fixed, split S2
increased flow across the pulmonic valve (Choice D) due to shunting of blood from the left atrium to the right atrium results in the tixed S2 associated with an atrial septal defect. The pulmonic valve itself is typically norma!.
Tricuspid valve (Choice E) dysfunction results in tricuspid regurgitation, which presents as a hoiosystolic murmur best heard in the left lower sternal border.
Educational Objective: A fixed, widely split S2 is characteristic of an atrial septal defect due to increased blood flow through the pulmonic valve. Severe defects can result in pulmonary hypertension and development of Eisenmenger syndrome overtime, with
reversal of the left to right shunt.
o © 0
* 0
*
K
Previous Next Score Report Lab Values Calculator Help Pause
Exam Section 1: Item 28 ol 50 National Board of Metlicai Examiners^
Comprehensive Basic Science Self-Assessment
y
MJW
_:
4
*
m
i «
-
V > I
LQJI. fc *
L
'
.i
i
•HI
"
T^W.
JP
TP r
1
*1
? .£
p
. L
s'm
-
f i
“
- —1
'
A
. >CJ *
-c fc
“VO i.
..I
p
r
SC" fi
J
p
Ol
IT
*
r £ T
m
a
.-i
- E
1
i
a p
tr
h.
d
k
^p
«J- »
R
a
I
i
a
.
a.
£
"a J
-K
28. A 53-year-old man has had progressive difficulty swallowing for the past 3 months. He has a 10-year history of heartburn with esophageal regurgitation of gastric contents. Tissue obtained on biopsy of the lower third of the esophagus is shown. Which of the
following best describes the nature of this lesion?
A } Basal zone hyperplasia of submucosal glands
B) Intestinal metaplasia of squamous epithelium
C ) Malignant transformation of epithelium into squamous carcinoma
D ) Squamous metaplasia of submucosal glands
Correct Answer: B.
Jntestina1
metaplasia of squamous epithelium in the esophagus, also known as Barrett esophagus, can be a consequence of prolonged gastroesophageai reflux disease (GERD): which occurs when acidic gastric contents reflux backward through the lower
esophageal sphincter into the esophagus. The mucosa of the esophagus is comprised of squamous epithefium and does not traditionally encounter such an acidic environment. Constant exposure to acidic intraluminal contents induces a change in cell type from
squamous epithelium to the columnar glandular epithelium found in the intestines as an adaptive response. These metaplastic cells will exhibit a brush border and goblet cells. Metaplasia can eventually lead to dysplasia, which is premalignant. Patients with
confirmed Barrett esophagus should be evaluated at regular intervals determined by the presence and/or grade of dysplasia. Treatment involves ablation of the dysplastic cells via endoscopy and management of the underlying GERD with a proton pump inhibitor,
dietary modification, and smoking cessation.
Incorrect Answers: A, C; and D.
Basal zone hyperplasia of submucosal glands (Choice A) is not the pathologic change observed in Barrett esophagus, although submucosal gland secretions do neutralize acidic luminai contents. They also lubricate the esophagus which allows for the food bolus
to pass.
Malignant transformation of epithelium into squamous carcinoma (Choice C) occurs with esophageal squamous carcinoma, which is more common in patients who consume alcohol and smoke cigarettes. Barrett esophagus primarify predisposes to
adenocarcinoma, not to squamous carcinoma.
Squamous metaplasia of submucosal glands (Choice D) is also associated with the development of esophageal adenocarcinoma. Submucosal glands contain progenitor cells that may play a role in the pathogenesis of dysplasia as these progenitor cells serve as
a source of potentially dysplastic or neoplastic cells, however, the pathophysiology of Barrett esophagus involves intestinal metaplasia.
Educational Objective: Barrett esophagus develops in individuals with chronic GERD and is histologically characterized by intestinal metaplasia whereby the normal squamous epithelium is replaced by columnar epithelium. Overtime,, dysplasia can develop,
predisposing to esophageal adenocarcinoma.
O 0 m (E
T
Previous Next Scone Report Lab Values Calculator Help Pause
Exam Section 1: Item 29 of 50 National Board of Medical Examiners^
Comprehensive Basic Science Self-Assessment
Y 29. A 50-year-old man comes to the emergency department because of a 2-week history of progressive shortness of breath. His pulse is 90/min, respirations are 26/mrn. and blood pressure is 120/80 mm Hg. Physical examination shows no other abnormalities.
Laboratory studies show:
Arterial Pco2
Arterial Po2
Arterial 02content
Mixed venous Po2
Mixed venous 02content 8 vol% (N=10%-16%)
30 mm Hg
96 mm Hg
12 vol% (N=17%-21%)
36 mm Hg
Which of the following is the most likety explanation for these findings?
A) Anemia
B) Drug-induced alveolar hypoventilation
C ) Residence at a high altitude
D ) Severe regional mismatching of alveolar ventilation and pulmonary capillary perfusion
E) Voluntary hyperventillation
Correct Answer: A.
The differentia! for dyspnea is broad and encompasses a range of disorders that involve impaired delivery of oxygen to tissue and/or reduced elimination of carbon dioxide from the body. The laboratory studies in this case indicates a reduced arterial and venous
oxygen content. The oxygen content of the blood is a function of the oxygen carrying capacity (essentially the hemoglobin concentration), percent saturation of hemoglobin, and partial pressure of dissolved molecular oxygen (Po
^ The equation to compute
oxygen content is thus: Oxygen content = 1.34*[Hemaglobin]*(Arterial Oxygen Saturation) + G.003*(Arterial PoJ. The amount of dissolved oxygen is negligible compared to the oxygen transported by hemoglobin. The patient in this case has reduced oxygen
content in the arterial and mixed venous circulation with a normal arterial Po2 In the absence of a hemoglobinopathy (eg. methemoglobinemia, carboxyhemoglobinemia), the patient's oxygen saturation is expected to be normal. The most likely diagnosis is
anemia with a decreased hemoglobin concentration. The arteriai Pco2 is decreased indicating hyperventilation, which is expected in the setting of decreased oxygen delivery to tissue.
incorrect Answers: B, C: D: and E.
Drug-induced alveolar hypoventilation (Choice B) would result in an increased arterial Pco
^
Potential etiologies include central nervous system depressants such as opioid analgesics and benzodiazepines.
Residence at a high altitude (Choice C) would be expected to result in a decreased Po2 with adaptive changes that maintain an adequate oxygen carrying capacity. These changes include secondary erythrocytosis with increased hemoglobin concentration and
increased levels of 2,3-bisphosphoglyceric acid: which stabilizes the deoxygenated state of hemoglobin and promotes increased oxygen reiease to tissue.
Severe regional mismatching of alveolar ventilation and pulmonary capillary perfusion (Choice D) occurs when either ventilation or perfusion to a region of lung is impaired. An example is pulmonary embolism, in which a region of ventilated lung has obstructed
blood flow.
Voluntary hyperventilation (Choice E) results in a respiratory alkalosis, with a decreased Pco
^
. Symptoms include dizziness, weakness, and syncope. The oxygen carrying capacity of the blood would not be reduced in the absence of other factors
Educational Objective: Delivery of oxygen to tissue is largely dependent on the hemoglobin concentration as it is the primary transporter of oxygen in the blood. Other contributing factors include the oxygen saturation of hemoglobin, and, less Significantly, the
partial pressure of dissolved molecular oxygen.
O 0
Previous Next Scone Report Lab Values Calculator Help Pause
Exam Section 1: Item 30 ol 50 National Board of Medical Examiners^
Comprehensive Basic Science Self-Assessment
Y 30. An investigator is studying the effects of triiodothyronine (TJ and thyroxine (TJin hepatocytes in an experimental animal model. Which of the following best describes the action of these thyroid hormones on this target tissue?
A ) Both T3 and T4 bind to the melanocortin 2 receptor on the cell surface
B) Both T5 and T4 enter the nucleus
C ) T3 is converted to
I
4 in the cytosol
D ) Thyroid hormone receptors preferentially bind T4 over T3
Correct Answer: B.
Both T3 (triiodothyronine) and T4 (thyroxine) are hormones that act on nuclear receptors, requiring them to enter the target ceil to exert effects. Unlike other lipophilic hormones, thyroid hormones contain charged amino acids that prevent passive diffusion across the
cellular membrane and thus enter by facilitated diffusion. Thyroid hormone transporters transport both T3 and T4 into the cell to reach their receptors. The thyroid hormone receptors are nuclear receptors that contain DNA-binding domains. Nuclear receptors can
initially be in either the cytosol or nucleus. Once nuclear receptors bind their respective hormones: they translocate into the nucleus, if not already there, where they act as DNA transcription factors to regulate the expression of target genes.
Incorrect Answers: A. C; and D.
Binding to the melanocortin 2 receptor on the cell surface (Choice A) does not occur with either T3 orT4. The melanocortin 2 receptor is also known as the adrenocorticotropic hormone (AGTH) receptor and is specific for ACTH. This receptor is a G protein-coupled
receptor and does not actively transport ACTH inside the cell.
Conversion of T3 to T4 in the cytosoi (Choice C) does not occur. T4 is the less active form of thyroid hormone and is converted to T3 In target cells. Once in the cell nucleus, T3 preferentially binds the receptor with greater affinity than T4 (Choice D): although both
hormones are capable of binding and activating the receptor.
Educational Objective: T3 and T4 act on nuclear receptors, requiring them to enter the target cell to exert effects. Unlike other lipophilic hormones, thyroid hormones contain charged amino acids that prevent passive diffusion across the cellular membrane. Thyroid
hormone transporters transport both T3 and T4 into the cell to reach their receptors.
0 0 # 0 f*
Previous Next Score Report Lab Values Calculator Help Pause
Ureteral Calculus Pain Relief
Ureteral Calculus Pain Relief
Ureteral Calculus Pain Relief
Ureteral Calculus Pain Relief
Ureteral Calculus Pain Relief
Ureteral Calculus Pain Relief
Ureteral Calculus Pain Relief
Ureteral Calculus Pain Relief
Ureteral Calculus Pain Relief
Ureteral Calculus Pain Relief
Ureteral Calculus Pain Relief
Ureteral Calculus Pain Relief
Ureteral Calculus Pain Relief
Ureteral Calculus Pain Relief
Ureteral Calculus Pain Relief
Ureteral Calculus Pain Relief
Ureteral Calculus Pain Relief
Ureteral Calculus Pain Relief
Ureteral Calculus Pain Relief
Ureteral Calculus Pain Relief
Ureteral Calculus Pain Relief
Ureteral Calculus Pain Relief
Ureteral Calculus Pain Relief
Ureteral Calculus Pain Relief
Ureteral Calculus Pain Relief
Ureteral Calculus Pain Relief
Ureteral Calculus Pain Relief
Ureteral Calculus Pain Relief
Ureteral Calculus Pain Relief
Ureteral Calculus Pain Relief
Ureteral Calculus Pain Relief
Ureteral Calculus Pain Relief
Ureteral Calculus Pain Relief
Ureteral Calculus Pain Relief
Ureteral Calculus Pain Relief
Ureteral Calculus Pain Relief
Ureteral Calculus Pain Relief
Ureteral Calculus Pain Relief
Ureteral Calculus Pain Relief
Ureteral Calculus Pain Relief
Ureteral Calculus Pain Relief
Ureteral Calculus Pain Relief
Ureteral Calculus Pain Relief
Ureteral Calculus Pain Relief
Ureteral Calculus Pain Relief
Ureteral Calculus Pain Relief
Ureteral Calculus Pain Relief
Ureteral Calculus Pain Relief
Ureteral Calculus Pain Relief
Ureteral Calculus Pain Relief
Ureteral Calculus Pain Relief
Ureteral Calculus Pain Relief
Ureteral Calculus Pain Relief
Ureteral Calculus Pain Relief
Ureteral Calculus Pain Relief
Ureteral Calculus Pain Relief
Ureteral Calculus Pain Relief
Ureteral Calculus Pain Relief
Ureteral Calculus Pain Relief
Ureteral Calculus Pain Relief
Ureteral Calculus Pain Relief
Ureteral Calculus Pain Relief
Ureteral Calculus Pain Relief
Ureteral Calculus Pain Relief
Ureteral Calculus Pain Relief
Ureteral Calculus Pain Relief
Ureteral Calculus Pain Relief
Ureteral Calculus Pain Relief
Ureteral Calculus Pain Relief
Ureteral Calculus Pain Relief
Ureteral Calculus Pain Relief
Ureteral Calculus Pain Relief
Ureteral Calculus Pain Relief
Ureteral Calculus Pain Relief
Ureteral Calculus Pain Relief
Ureteral Calculus Pain Relief
Ureteral Calculus Pain Relief
Ureteral Calculus Pain Relief
Ureteral Calculus Pain Relief
Ureteral Calculus Pain Relief
Ureteral Calculus Pain Relief
Ureteral Calculus Pain Relief
Ureteral Calculus Pain Relief
Ureteral Calculus Pain Relief
Ureteral Calculus Pain Relief
Ureteral Calculus Pain Relief
Ureteral Calculus Pain Relief
Ureteral Calculus Pain Relief
Ureteral Calculus Pain Relief
Ureteral Calculus Pain Relief
Ureteral Calculus Pain Relief
Ureteral Calculus Pain Relief
Ureteral Calculus Pain Relief
Ureteral Calculus Pain Relief
Ureteral Calculus Pain Relief
Ureteral Calculus Pain Relief
Ureteral Calculus Pain Relief
Ureteral Calculus Pain Relief
Ureteral Calculus Pain Relief
Ureteral Calculus Pain Relief
Ureteral Calculus Pain Relief
Ureteral Calculus Pain Relief
Ureteral Calculus Pain Relief
Ureteral Calculus Pain Relief
Ureteral Calculus Pain Relief
Ureteral Calculus Pain Relief
Ureteral Calculus Pain Relief
Ureteral Calculus Pain Relief
Ureteral Calculus Pain Relief
Ureteral Calculus Pain Relief
Ureteral Calculus Pain Relief
Ureteral Calculus Pain Relief
Ureteral Calculus Pain Relief
Ureteral Calculus Pain Relief
Ureteral Calculus Pain Relief
Ureteral Calculus Pain Relief
Ureteral Calculus Pain Relief
Ureteral Calculus Pain Relief
Ureteral Calculus Pain Relief
Ureteral Calculus Pain Relief
Ureteral Calculus Pain Relief
Ureteral Calculus Pain Relief
Ureteral Calculus Pain Relief
Ureteral Calculus Pain Relief
Ureteral Calculus Pain Relief
Ureteral Calculus Pain Relief
Ureteral Calculus Pain Relief
Ureteral Calculus Pain Relief
Ureteral Calculus Pain Relief
Ureteral Calculus Pain Relief
Ureteral Calculus Pain Relief
Ureteral Calculus Pain Relief
Ureteral Calculus Pain Relief
Ureteral Calculus Pain Relief
Ureteral Calculus Pain Relief
Ureteral Calculus Pain Relief
Ureteral Calculus Pain Relief
Ureteral Calculus Pain Relief
Ureteral Calculus Pain Relief
Ureteral Calculus Pain Relief
Ureteral Calculus Pain Relief
Ureteral Calculus Pain Relief
Ureteral Calculus Pain Relief
Ureteral Calculus Pain Relief
Ureteral Calculus Pain Relief
Ureteral Calculus Pain Relief
Ureteral Calculus Pain Relief
Ureteral Calculus Pain Relief
Ureteral Calculus Pain Relief
Ureteral Calculus Pain Relief
Ureteral Calculus Pain Relief
Ureteral Calculus Pain Relief
Ureteral Calculus Pain Relief
Ureteral Calculus Pain Relief
Ureteral Calculus Pain Relief
Ureteral Calculus Pain Relief
Ureteral Calculus Pain Relief
Ureteral Calculus Pain Relief
Ureteral Calculus Pain Relief
Ureteral Calculus Pain Relief
Ureteral Calculus Pain Relief
Ureteral Calculus Pain Relief
Ureteral Calculus Pain Relief
Ureteral Calculus Pain Relief
Ureteral Calculus Pain Relief
Ureteral Calculus Pain Relief
Ureteral Calculus Pain Relief
Ureteral Calculus Pain Relief
Ureteral Calculus Pain Relief
Ureteral Calculus Pain Relief

More Related Content

Similar to Ureteral Calculus Pain Relief

Pitfalls in orthopaedics
Pitfalls in orthopaedicsPitfalls in orthopaedics
Pitfalls in orthopaedicsPramod Mahender
 
Abdominal trauma
Abdominal traumaAbdominal trauma
Abdominal traumawanted1361
 
Invasive Options Kimh Invasive 2009
Invasive Options Kimh Invasive 2009Invasive Options Kimh Invasive 2009
Invasive Options Kimh Invasive 2009Tim Bushnell
 
01 blunt abdominal trauma
01 blunt abdominal trauma01 blunt abdominal trauma
01 blunt abdominal traumaDang Thanh Tuan
 
Mod 3 case 2022
Mod 3 case 2022Mod 3 case 2022
Mod 3 case 2022vetindex4
 
Inguinal Hernia Management, Presentation, by Dr. Shabir Ahmad
Inguinal Hernia Management, Presentation, by Dr. Shabir Ahmad Inguinal Hernia Management, Presentation, by Dr. Shabir Ahmad
Inguinal Hernia Management, Presentation, by Dr. Shabir Ahmad Shabir Ahmad
 
Spinal cord disorders
Spinal cord disordersSpinal cord disorders
Spinal cord disordersAmr Hassan
 
5. Inguinal Hernia & Surgical Anatomy.pptx
5. Inguinal Hernia & Surgical Anatomy.pptx5. Inguinal Hernia & Surgical Anatomy.pptx
5. Inguinal Hernia & Surgical Anatomy.pptxMamoon Saleh
 
Neurosurgical Intracranial Infections - FINAL 10-17-23.pptx
Neurosurgical Intracranial Infections - FINAL 10-17-23.pptxNeurosurgical Intracranial Infections - FINAL 10-17-23.pptx
Neurosurgical Intracranial Infections - FINAL 10-17-23.pptxSean M. Fox
 
fecal incontinence
fecal incontinencefecal incontinence
fecal incontinencegom3a2010
 
Hernia and herniorrhaphy
Hernia and herniorrhaphyHernia and herniorrhaphy
Hernia and herniorrhaphyAdams Inusah
 
23205016
2320501623205016
23205016radgirl
 
MRCS Anatomy MCQs .pdf
MRCS Anatomy MCQs .pdfMRCS Anatomy MCQs .pdf
MRCS Anatomy MCQs .pdfKhaula1
 
Brachial Plexus injuries (2).pptx
Brachial Plexus injuries (2).pptxBrachial Plexus injuries (2).pptx
Brachial Plexus injuries (2).pptxprashanthNaik44
 
Cpc Icd-9-Cm Coding Essay
Cpc Icd-9-Cm Coding EssayCpc Icd-9-Cm Coding Essay
Cpc Icd-9-Cm Coding EssayDivya Watson
 

Similar to Ureteral Calculus Pain Relief (20)

Pitfalls in orthopaedics
Pitfalls in orthopaedicsPitfalls in orthopaedics
Pitfalls in orthopaedics
 
Hernia
HerniaHernia
Hernia
 
Abdominal trauma
Abdominal traumaAbdominal trauma
Abdominal trauma
 
Invasive Options Kimh Invasive 2009
Invasive Options Kimh Invasive 2009Invasive Options Kimh Invasive 2009
Invasive Options Kimh Invasive 2009
 
01 blunt abdominal trauma
01 blunt abdominal trauma01 blunt abdominal trauma
01 blunt abdominal trauma
 
Mod 3 case 2022
Mod 3 case 2022Mod 3 case 2022
Mod 3 case 2022
 
Hernias 2.ppt
Hernias 2.pptHernias 2.ppt
Hernias 2.ppt
 
Inguinal Hernia Management, Presentation, by Dr. Shabir Ahmad
Inguinal Hernia Management, Presentation, by Dr. Shabir Ahmad Inguinal Hernia Management, Presentation, by Dr. Shabir Ahmad
Inguinal Hernia Management, Presentation, by Dr. Shabir Ahmad
 
Spinal cord disorders
Spinal cord disordersSpinal cord disorders
Spinal cord disorders
 
5. Inguinal Hernia & Surgical Anatomy.pptx
5. Inguinal Hernia & Surgical Anatomy.pptx5. Inguinal Hernia & Surgical Anatomy.pptx
5. Inguinal Hernia & Surgical Anatomy.pptx
 
Thoracic outlet syndrome
Thoracic outlet syndromeThoracic outlet syndrome
Thoracic outlet syndrome
 
Neurosurgical Intracranial Infections - FINAL 10-17-23.pptx
Neurosurgical Intracranial Infections - FINAL 10-17-23.pptxNeurosurgical Intracranial Infections - FINAL 10-17-23.pptx
Neurosurgical Intracranial Infections - FINAL 10-17-23.pptx
 
fecal incontinence
fecal incontinencefecal incontinence
fecal incontinence
 
Hernia and herniorrhaphy
Hernia and herniorrhaphyHernia and herniorrhaphy
Hernia and herniorrhaphy
 
Ankylosing spondylitis. (ben)
Ankylosing spondylitis. (ben)Ankylosing spondylitis. (ben)
Ankylosing spondylitis. (ben)
 
23205016
2320501623205016
23205016
 
MRCS Anatomy MCQs .pdf
MRCS Anatomy MCQs .pdfMRCS Anatomy MCQs .pdf
MRCS Anatomy MCQs .pdf
 
Brachial Plexus injuries (2).pptx
Brachial Plexus injuries (2).pptxBrachial Plexus injuries (2).pptx
Brachial Plexus injuries (2).pptx
 
HERNIA-1.pptx
HERNIA-1.pptxHERNIA-1.pptx
HERNIA-1.pptx
 
Cpc Icd-9-Cm Coding Essay
Cpc Icd-9-Cm Coding EssayCpc Icd-9-Cm Coding Essay
Cpc Icd-9-Cm Coding Essay
 

Recently uploaded

Kesar Bagh Call Girl Price 9548273370 , Lucknow Call Girls Service
Kesar Bagh Call Girl Price 9548273370 , Lucknow Call Girls ServiceKesar Bagh Call Girl Price 9548273370 , Lucknow Call Girls Service
Kesar Bagh Call Girl Price 9548273370 , Lucknow Call Girls Servicemakika9823
 
(👑VVIP ISHAAN ) Russian Call Girls Service Navi Mumbai🖕9920874524🖕Independent...
(👑VVIP ISHAAN ) Russian Call Girls Service Navi Mumbai🖕9920874524🖕Independent...(👑VVIP ISHAAN ) Russian Call Girls Service Navi Mumbai🖕9920874524🖕Independent...
(👑VVIP ISHAAN ) Russian Call Girls Service Navi Mumbai🖕9920874524🖕Independent...Taniya Sharma
 
(Rocky) Jaipur Call Girl - 9521753030 Escorts Service 50% Off with Cash ON De...
(Rocky) Jaipur Call Girl - 9521753030 Escorts Service 50% Off with Cash ON De...(Rocky) Jaipur Call Girl - 9521753030 Escorts Service 50% Off with Cash ON De...
(Rocky) Jaipur Call Girl - 9521753030 Escorts Service 50% Off with Cash ON De...indiancallgirl4rent
 
Call Girls Service Bellary Road Just Call 7001305949 Enjoy College Girls Service
Call Girls Service Bellary Road Just Call 7001305949 Enjoy College Girls ServiceCall Girls Service Bellary Road Just Call 7001305949 Enjoy College Girls Service
Call Girls Service Bellary Road Just Call 7001305949 Enjoy College Girls Servicenarwatsonia7
 
💎VVIP Kolkata Call Girls Parganas🩱7001035870🩱Independent Girl ( Ac Rooms Avai...
💎VVIP Kolkata Call Girls Parganas🩱7001035870🩱Independent Girl ( Ac Rooms Avai...💎VVIP Kolkata Call Girls Parganas🩱7001035870🩱Independent Girl ( Ac Rooms Avai...
💎VVIP Kolkata Call Girls Parganas🩱7001035870🩱Independent Girl ( Ac Rooms Avai...Taniya Sharma
 
VIP Call Girls Pune Vani 9907093804 Short 1500 Night 6000 Best call girls Ser...
VIP Call Girls Pune Vani 9907093804 Short 1500 Night 6000 Best call girls Ser...VIP Call Girls Pune Vani 9907093804 Short 1500 Night 6000 Best call girls Ser...
VIP Call Girls Pune Vani 9907093804 Short 1500 Night 6000 Best call girls Ser...Miss joya
 
VIP Call Girls Pune Sanjana 9907093804 Short 1500 Night 6000 Best call girls ...
VIP Call Girls Pune Sanjana 9907093804 Short 1500 Night 6000 Best call girls ...VIP Call Girls Pune Sanjana 9907093804 Short 1500 Night 6000 Best call girls ...
VIP Call Girls Pune Sanjana 9907093804 Short 1500 Night 6000 Best call girls ...Miss joya
 
Call Girls Service Surat Samaira ❤️🍑 8250192130 👄 Independent Escort Service ...
Call Girls Service Surat Samaira ❤️🍑 8250192130 👄 Independent Escort Service ...Call Girls Service Surat Samaira ❤️🍑 8250192130 👄 Independent Escort Service ...
Call Girls Service Surat Samaira ❤️🍑 8250192130 👄 Independent Escort Service ...CALL GIRLS
 
Bangalore Call Girls Marathahalli 📞 9907093804 High Profile Service 100% Safe
Bangalore Call Girls Marathahalli 📞 9907093804 High Profile Service 100% SafeBangalore Call Girls Marathahalli 📞 9907093804 High Profile Service 100% Safe
Bangalore Call Girls Marathahalli 📞 9907093804 High Profile Service 100% Safenarwatsonia7
 
VIP Call Girls Pune Vrinda 9907093804 Short 1500 Night 6000 Best call girls S...
VIP Call Girls Pune Vrinda 9907093804 Short 1500 Night 6000 Best call girls S...VIP Call Girls Pune Vrinda 9907093804 Short 1500 Night 6000 Best call girls S...
VIP Call Girls Pune Vrinda 9907093804 Short 1500 Night 6000 Best call girls S...Miss joya
 
Aspirin presentation slides by Dr. Rewas Ali
Aspirin presentation slides by Dr. Rewas AliAspirin presentation slides by Dr. Rewas Ali
Aspirin presentation slides by Dr. Rewas AliRewAs ALI
 
Call Girls Service Pune Vaishnavi 9907093804 Short 1500 Night 6000 Best call ...
Call Girls Service Pune Vaishnavi 9907093804 Short 1500 Night 6000 Best call ...Call Girls Service Pune Vaishnavi 9907093804 Short 1500 Night 6000 Best call ...
Call Girls Service Pune Vaishnavi 9907093804 Short 1500 Night 6000 Best call ...Miss joya
 
Bangalore Call Girls Hebbal Kempapura Number 7001035870 Meetin With Bangalor...
Bangalore Call Girls Hebbal Kempapura Number 7001035870  Meetin With Bangalor...Bangalore Call Girls Hebbal Kempapura Number 7001035870  Meetin With Bangalor...
Bangalore Call Girls Hebbal Kempapura Number 7001035870 Meetin With Bangalor...narwatsonia7
 
Russian Escorts Girls Nehru Place ZINATHI 🔝9711199012 ☪ 24/7 Call Girls Delhi
Russian Escorts Girls  Nehru Place ZINATHI 🔝9711199012 ☪ 24/7 Call Girls DelhiRussian Escorts Girls  Nehru Place ZINATHI 🔝9711199012 ☪ 24/7 Call Girls Delhi
Russian Escorts Girls Nehru Place ZINATHI 🔝9711199012 ☪ 24/7 Call Girls DelhiAlinaDevecerski
 
Call Girls Cuttack Just Call 9907093804 Top Class Call Girl Service Available
Call Girls Cuttack Just Call 9907093804 Top Class Call Girl Service AvailableCall Girls Cuttack Just Call 9907093804 Top Class Call Girl Service Available
Call Girls Cuttack Just Call 9907093804 Top Class Call Girl Service AvailableDipal Arora
 
Call Girl Coimbatore Prisha☎️ 8250192130 Independent Escort Service Coimbatore
Call Girl Coimbatore Prisha☎️  8250192130 Independent Escort Service CoimbatoreCall Girl Coimbatore Prisha☎️  8250192130 Independent Escort Service Coimbatore
Call Girl Coimbatore Prisha☎️ 8250192130 Independent Escort Service Coimbatorenarwatsonia7
 
VIP Mumbai Call Girls Hiranandani Gardens Just Call 9920874524 with A/C Room ...
VIP Mumbai Call Girls Hiranandani Gardens Just Call 9920874524 with A/C Room ...VIP Mumbai Call Girls Hiranandani Gardens Just Call 9920874524 with A/C Room ...
VIP Mumbai Call Girls Hiranandani Gardens Just Call 9920874524 with A/C Room ...Garima Khatri
 
Bangalore Call Girls Nelamangala Number 7001035870 Meetin With Bangalore Esc...
Bangalore Call Girls Nelamangala Number 7001035870  Meetin With Bangalore Esc...Bangalore Call Girls Nelamangala Number 7001035870  Meetin With Bangalore Esc...
Bangalore Call Girls Nelamangala Number 7001035870 Meetin With Bangalore Esc...narwatsonia7
 
High Profile Call Girls Coimbatore Saanvi☎️ 8250192130 Independent Escort Se...
High Profile Call Girls Coimbatore Saanvi☎️  8250192130 Independent Escort Se...High Profile Call Girls Coimbatore Saanvi☎️  8250192130 Independent Escort Se...
High Profile Call Girls Coimbatore Saanvi☎️ 8250192130 Independent Escort Se...narwatsonia7
 
CALL ON ➥9907093804 🔝 Call Girls Hadapsar ( Pune) Girls Service
CALL ON ➥9907093804 🔝 Call Girls Hadapsar ( Pune)  Girls ServiceCALL ON ➥9907093804 🔝 Call Girls Hadapsar ( Pune)  Girls Service
CALL ON ➥9907093804 🔝 Call Girls Hadapsar ( Pune) Girls ServiceMiss joya
 

Recently uploaded (20)

Kesar Bagh Call Girl Price 9548273370 , Lucknow Call Girls Service
Kesar Bagh Call Girl Price 9548273370 , Lucknow Call Girls ServiceKesar Bagh Call Girl Price 9548273370 , Lucknow Call Girls Service
Kesar Bagh Call Girl Price 9548273370 , Lucknow Call Girls Service
 
(👑VVIP ISHAAN ) Russian Call Girls Service Navi Mumbai🖕9920874524🖕Independent...
(👑VVIP ISHAAN ) Russian Call Girls Service Navi Mumbai🖕9920874524🖕Independent...(👑VVIP ISHAAN ) Russian Call Girls Service Navi Mumbai🖕9920874524🖕Independent...
(👑VVIP ISHAAN ) Russian Call Girls Service Navi Mumbai🖕9920874524🖕Independent...
 
(Rocky) Jaipur Call Girl - 9521753030 Escorts Service 50% Off with Cash ON De...
(Rocky) Jaipur Call Girl - 9521753030 Escorts Service 50% Off with Cash ON De...(Rocky) Jaipur Call Girl - 9521753030 Escorts Service 50% Off with Cash ON De...
(Rocky) Jaipur Call Girl - 9521753030 Escorts Service 50% Off with Cash ON De...
 
Call Girls Service Bellary Road Just Call 7001305949 Enjoy College Girls Service
Call Girls Service Bellary Road Just Call 7001305949 Enjoy College Girls ServiceCall Girls Service Bellary Road Just Call 7001305949 Enjoy College Girls Service
Call Girls Service Bellary Road Just Call 7001305949 Enjoy College Girls Service
 
💎VVIP Kolkata Call Girls Parganas🩱7001035870🩱Independent Girl ( Ac Rooms Avai...
💎VVIP Kolkata Call Girls Parganas🩱7001035870🩱Independent Girl ( Ac Rooms Avai...💎VVIP Kolkata Call Girls Parganas🩱7001035870🩱Independent Girl ( Ac Rooms Avai...
💎VVIP Kolkata Call Girls Parganas🩱7001035870🩱Independent Girl ( Ac Rooms Avai...
 
VIP Call Girls Pune Vani 9907093804 Short 1500 Night 6000 Best call girls Ser...
VIP Call Girls Pune Vani 9907093804 Short 1500 Night 6000 Best call girls Ser...VIP Call Girls Pune Vani 9907093804 Short 1500 Night 6000 Best call girls Ser...
VIP Call Girls Pune Vani 9907093804 Short 1500 Night 6000 Best call girls Ser...
 
VIP Call Girls Pune Sanjana 9907093804 Short 1500 Night 6000 Best call girls ...
VIP Call Girls Pune Sanjana 9907093804 Short 1500 Night 6000 Best call girls ...VIP Call Girls Pune Sanjana 9907093804 Short 1500 Night 6000 Best call girls ...
VIP Call Girls Pune Sanjana 9907093804 Short 1500 Night 6000 Best call girls ...
 
Call Girls Service Surat Samaira ❤️🍑 8250192130 👄 Independent Escort Service ...
Call Girls Service Surat Samaira ❤️🍑 8250192130 👄 Independent Escort Service ...Call Girls Service Surat Samaira ❤️🍑 8250192130 👄 Independent Escort Service ...
Call Girls Service Surat Samaira ❤️🍑 8250192130 👄 Independent Escort Service ...
 
Bangalore Call Girls Marathahalli 📞 9907093804 High Profile Service 100% Safe
Bangalore Call Girls Marathahalli 📞 9907093804 High Profile Service 100% SafeBangalore Call Girls Marathahalli 📞 9907093804 High Profile Service 100% Safe
Bangalore Call Girls Marathahalli 📞 9907093804 High Profile Service 100% Safe
 
VIP Call Girls Pune Vrinda 9907093804 Short 1500 Night 6000 Best call girls S...
VIP Call Girls Pune Vrinda 9907093804 Short 1500 Night 6000 Best call girls S...VIP Call Girls Pune Vrinda 9907093804 Short 1500 Night 6000 Best call girls S...
VIP Call Girls Pune Vrinda 9907093804 Short 1500 Night 6000 Best call girls S...
 
Aspirin presentation slides by Dr. Rewas Ali
Aspirin presentation slides by Dr. Rewas AliAspirin presentation slides by Dr. Rewas Ali
Aspirin presentation slides by Dr. Rewas Ali
 
Call Girls Service Pune Vaishnavi 9907093804 Short 1500 Night 6000 Best call ...
Call Girls Service Pune Vaishnavi 9907093804 Short 1500 Night 6000 Best call ...Call Girls Service Pune Vaishnavi 9907093804 Short 1500 Night 6000 Best call ...
Call Girls Service Pune Vaishnavi 9907093804 Short 1500 Night 6000 Best call ...
 
Bangalore Call Girls Hebbal Kempapura Number 7001035870 Meetin With Bangalor...
Bangalore Call Girls Hebbal Kempapura Number 7001035870  Meetin With Bangalor...Bangalore Call Girls Hebbal Kempapura Number 7001035870  Meetin With Bangalor...
Bangalore Call Girls Hebbal Kempapura Number 7001035870 Meetin With Bangalor...
 
Russian Escorts Girls Nehru Place ZINATHI 🔝9711199012 ☪ 24/7 Call Girls Delhi
Russian Escorts Girls  Nehru Place ZINATHI 🔝9711199012 ☪ 24/7 Call Girls DelhiRussian Escorts Girls  Nehru Place ZINATHI 🔝9711199012 ☪ 24/7 Call Girls Delhi
Russian Escorts Girls Nehru Place ZINATHI 🔝9711199012 ☪ 24/7 Call Girls Delhi
 
Call Girls Cuttack Just Call 9907093804 Top Class Call Girl Service Available
Call Girls Cuttack Just Call 9907093804 Top Class Call Girl Service AvailableCall Girls Cuttack Just Call 9907093804 Top Class Call Girl Service Available
Call Girls Cuttack Just Call 9907093804 Top Class Call Girl Service Available
 
Call Girl Coimbatore Prisha☎️ 8250192130 Independent Escort Service Coimbatore
Call Girl Coimbatore Prisha☎️  8250192130 Independent Escort Service CoimbatoreCall Girl Coimbatore Prisha☎️  8250192130 Independent Escort Service Coimbatore
Call Girl Coimbatore Prisha☎️ 8250192130 Independent Escort Service Coimbatore
 
VIP Mumbai Call Girls Hiranandani Gardens Just Call 9920874524 with A/C Room ...
VIP Mumbai Call Girls Hiranandani Gardens Just Call 9920874524 with A/C Room ...VIP Mumbai Call Girls Hiranandani Gardens Just Call 9920874524 with A/C Room ...
VIP Mumbai Call Girls Hiranandani Gardens Just Call 9920874524 with A/C Room ...
 
Bangalore Call Girls Nelamangala Number 7001035870 Meetin With Bangalore Esc...
Bangalore Call Girls Nelamangala Number 7001035870  Meetin With Bangalore Esc...Bangalore Call Girls Nelamangala Number 7001035870  Meetin With Bangalore Esc...
Bangalore Call Girls Nelamangala Number 7001035870 Meetin With Bangalore Esc...
 
High Profile Call Girls Coimbatore Saanvi☎️ 8250192130 Independent Escort Se...
High Profile Call Girls Coimbatore Saanvi☎️  8250192130 Independent Escort Se...High Profile Call Girls Coimbatore Saanvi☎️  8250192130 Independent Escort Se...
High Profile Call Girls Coimbatore Saanvi☎️ 8250192130 Independent Escort Se...
 
CALL ON ➥9907093804 🔝 Call Girls Hadapsar ( Pune) Girls Service
CALL ON ➥9907093804 🔝 Call Girls Hadapsar ( Pune)  Girls ServiceCALL ON ➥9907093804 🔝 Call Girls Hadapsar ( Pune)  Girls Service
CALL ON ➥9907093804 🔝 Call Girls Hadapsar ( Pune) Girls Service
 

Ureteral Calculus Pain Relief

  • 1. Exam Section 1: Item 1 of 50 National Board of Medical Examiners^ Comprehensive Basic Science Self-Assessment Y 1. A 25-year-old man is brought to the emergency department because of severe abdominal pain, nausea: and vomiting for 1 hour. The pain originates in the left flank and radiates to his groin. His pulse is 100/min, respirations are 18/minp and blood pressure is 150/100 mm Hg. Physical examination shows tenderness of the left flank and the left lower quadrant of the abdomen. Bowel sounds are mildly hypoactive. Test of the stool for occult blood is negative. Which of the following best explains these findings? A ) Colon neoplasm B) Diverticulitis C ) Epididymitis D ) Renal infarction E) Torsion of the testis F) Ureteral calculus Correct Answer: F. Ureteral calculus typically presents with colicky unilateral flank pain radiating to the groin: and with gross or microscopic hematuria. Pain may be significant enough to trigger nausea, as in this case. The common types of urinary tract calculi are calcium oxalate or phosphate, ammonium magnesium phosphate: uric acid: and cystine. On urinalysis, red blood cells without casts are common. Fever dysuria; and pyuria would not be expected unless there was a concomitant infection. Treatment for ureteral calculus is symptomatic with pain control and nausea relief. Most ureteral calculi pass spontaneously after a period of observation for patients with well-control ed pain and no signs of sepsis or infection. Stone removal by shock wave ithotripsy or endoscopic removal is an option for patients requiring emergency therapy. It is also an option for patients with persistent obstruction, uncontrolled symptoms, or failure of stone progression. In general, stones smaller than 5 mm will pass without operative assistance. Obstructing stones may require temporary placement of a ureteral stent to prevent hydronephrosis and renal parenchymal injury. Incorrect Answers: A. B. Ca D, and E Colon neoplasm (Choice A) would be unlikely in an otherwise healthy young patient with no family history of polyposis syndromes and acute, severe, flank pain. It would typically present with insidious weight loss, anemia, constipation or blood per rectum. In addition, test for stool for occult blood is negative, making this diagnosis unlikely. Diverticulitis (Choice B) can present with left lower quadrant abdominal pain and tenderness but would be less abrupt in presentation and typically present with fever, diarrhea, and hyperactive bowel sounds. It would be unlikely to cause flank pair Epididymitis (Choice C) is a common cause of painful scrotal swelling and refers to acute infection and inflammation of the epididymis. In younger males, this is commonly secondary to sexually transmitted infections such as Chlamydia trachomatis or Neisseria gonorrhoeae. In older males, Escherichia coli is more common. Renai infarction (Choice D) can cause flank pain, nausea, and vomiting, and can be due to thromboembolic disease, renal artery dissection, or a hypercoagulable state. However, it is rare and ureteral calculus is mare common and likely in this patient. Torsion of the testis (Choice E) occurs when the testicle twists on the spermatic cord resulting in subsequent loss of testicular blood supply. Patients typically present with acute, severe testicular pain, swelling, and erythema. On physical examination, the testicle typically demonstrates an abnormal lie (eg, transverse), extreme tenderness to palpation, absent cremasteric reflex, and pain that does not improve with elevation of the scrotum (as it does in epididymitis). Educational Objective: Ureteral calculus typically presents with colicky, unilateral flank pain radiating to the groin, along with gross or microscopic hematuria. 0 0 <t= A Next Score Report Lab Values Calculator Help Pause
  • 2. Exam Section 1: Item 2 of 50 National Board of Medical Examiners^ Comprehensive Basic Science Self-Assessment Y 2. Which of the following types of sensory information is compromised by lesions of the structure at site X in the photograph shown? A } Conscious proprioception B) Pain sensation C) Two-point discrimination D) Unconscious proprioception E) Vibration sense Correct Answer: D. The anterior lobe of the cerebellum (labeled X. pictured in cross-section as an arborized brain area posterior to the brainstem and anterior to the primary fissure of the cerebellum) mediates unconscious proprioception. The anterior lobe of the cerebellum receives information from the spinocerebellar tract about proprioception, or body position, that is gathered from muscle stretch and tension receptors on the ipsilatera! side of the body. This proprioceptive information is transmitted outside of conscious awareness. The deep cerebellar nuclei use this proprioceptive information to control motor learning, movement course changes, and balance. Damage to the anterior lobe of the cerebellum, which commonly occurs in chronic alcoholism, may lead to broad-based gait ataxia. Incorrect Answers: A. B: C. and E. Conscious proprioception (Choice A), two-point discrimination (Choice C), and vibration sense (Choice E) are mediated by the dorsal column-media lemniscus pathway, which relays this sensory information up the spinal cord to the thalamus and terminates in the primary sensory cortex in the parietal lobe. The cortex is a high-order brain area involved in several conscious brain functions, which reflects this pathway's mediation of the conscious (rather than unconscious) awareness of proprioception. Pain sensation (Choice B) is mediated by the spinothalamic pathway. The spinothalamic pathway transmits information about pain, temperature, and crude touch up the spina! cord to the thalamus, terminating in the primary sensory cortex. Educational Objective: The anterior lobe of the cerebellum mediates unconscious proprioception, whereas conscious proprioception is controlled by the dorsal column-medial lemniscus pathway. Lesions of the anterior lobe of the cerebellum can result in broad- based gait ataxia. 0 0 0 Previous Next Score Report Lab Values Calculator Help Pause
  • 3. Exam Section 1: Item 3 of 50 National Board of Medical Examiners^ Comprehensive Basic Science Self-Assessment Y 3. Moving the forearm against resistance from palm-down to palm-up (supination) position requires the use of which of the following muscles? A) Biceps brachii B) Brachialis C) Triceps D) Flexor carpi radialis E) Pronator teres Correct Answer: A. The biceps brachii muscle has two main actions, flexion of the elbow joint and supination of the forearm. The biceps brachii contains two proximal heads, with the short head attaching to the coracoid process of the scapula and the long head entering the shoulder joint and attaching to the supraglenoid tubercle. The distal biceps tendon inserts on the bicipital tuberosity of the proximal radius. Because of its orientation crossing the elbow joint, contraction of this muscle causes elbow flexion. Its eccentric insertion on the proximal radius allows for it to wind around the radius during pronation and unwind when contracted from around the proximal radius during supination. Incorrect Answers: B, C. D and E The brachialis muscle (Choice B) originates on the anterior surface of the humerus and crosses the elbow inserting on the tuberosity of the ulna. It does not wrap around the ulna and the ulna does not rotate. Because of this, it does not contribute to supination or pronation. The triceps muscle (Choice C) serves to extend the elbow joint. Proximally, it originates from the infraglenoid tubercle of the scapula (long head}, just proximal to the radial groove (lateral head), and just distal to the radial groove (medial head). Distally: it inserts on the olecranon process of the ulna. Contraction of this muscle extends the elbow and does not contribute to rotation. Mexor carpi radialis (Choice D) originates on the medial epicondyle of the humerus and inserts on the second and third metacarpal bones. This allows for flexion of the wrist. Pronator teres (Choice E) is a muscle of the proximal forearm that extends from the medial supracondylar ridge of the humerus and inserts on the lateral aspect of the radius. Contraction along this axis will promote pronation: not supination. Educational Objective: The biceps brachii muscle has two main functions. It serves to supinate the forearm through its winding mechanism around the proximal radius and bicipital tuberosity. It flexes the elbow as the muscle and tendon cross the elbow joint anteriorly. 0 0 & 0 0 Previous Next Score Report Lab Values Calculator Help Pause
  • 4. Exam Section 1: Item 4 of 50 National Board of Medical Examiners^ Comprehensive Basic Science Self-Assessment Y 4. A 35-year-old man is admitted to the hospita because of a 5-day history of fever and dyspnea He underwent a bone marrow transplantation S months ago; the procedure was complicated by severe graft-versus-host disease. His temperature is 33°C (100.4T) and respirations are 30/min. Scattered crackles are heard on auscultation of the chest. A chest x-ray shows patchy infiltrates A transbronchial biopsy specimen shows findings consistent with cytomegalovirus infection. Intravenous administration of ganciclovir is begun. This drug interferes with the function of which of the following enzymes? A) DNA polymerase B) Jntegrase C ) Reverse transcriptase D ) RNA polymerase E) Thymidine kinase Correct Answer: A. Cytomegalovirus (CMV), also known as human herpesvirus 5 (HHV-5); is an opportunistic infection commonly occurring in immunocompromised patients with solid-organ or allogeneic bone marrow transplantation severe ulcerative colitis, or HIV/AIDS infection. It can be transmitted through multiple modes, including sexuai contact urine, respiratory droplets: and to a fetus via the placenta. It can cause a variety of presentations, including mononucleosis in immunocompetent patients, and retinitis, esophagitis, and pneumonia in immunocompromised patients. Treatment for all human herpesvirus infections involves drugs that inhibit viral DNA polymerase, classically by guanosine analogs such as acyclovir valacyclovir, and famciclovir. Prior to exerting their antivira effects, mos' guanosine analogs must be phosphorytated by the viral enzyme thymidine kinase. They are then able to inhibit the viral DNA polymerase by terminating the nascent DNA chain during replication. These drugs are effective against herpes simplex virus and varicella zoster virus, weakly effective against Epstein-Barr virus, and not effective against CMV, which does not have the necessary thymidine kinase needed for phosphorylation. However, it does have the necessary phosphorylating enzyme to activate ganciclovir, another guanosine analog that inhibits DNA polymerase, and thus this is an effective anti-viral treatment for CMV Incorrect Answers: B. C, D, and E. Integrase inhibitors (Choice B) prevent integration of proviral DNA into the host genome. They are a component of the highly active anti-retrovirai therapy utilized in the treatment of HIV. Reverse transcriptase (Choice C) transcribes DNA from viral mRNA for incorporation into the host cell genome. This enzyme is inhibited by both nucleoside reverse transcriptase inhibitors, such as abacavir and didanosine. and non-nucleoside reverse transcriptase inhibitors, such as efavirenz and nevirapine. These medications are used to treat HIV infections. RNA polymerase (Choice D) is the enzyme that facilitates the conversion of DNA to RNA. It is the target of nucleotide and nucleoside polymerase inhibitors, which are utilized in the treatment of Hepatitis C infection. Thymidine kinase (Choice E) is a phosphorylating enzyme required for activation of the guanosine analogs acyclovir valacyclovir. and famciclovir. Development of a mutation in the viral thymidine kinase enzyme prevents drug phosphorylation and confers resistance to these medications. CMV does not contain the thymidine kinase enzyme. Educational Objective: Ganciclovir is a guanosine analog that inhibits DNA polymerase and is used to treat CMV . Like other guanosine analogs activation of this medication requires phosphorylation within the infected cell © © © Previous Next Score Report Lab Values Calculator Help Pause
  • 5. Exam Section 1: Item 5 of 50 National Board of Metlicai Examiners^ Comprehensive Basic Science Self-Assessment y 0 200- T3 rS E E CL 100- <t> c/> 0 25 27 1 Cycle (day) M = menstruation 5. The graph shows changes in serum estradiol concentration during a normal menstrual cycle. Which of the following ovarian cells is primarily responsible for the aromatization of androstenedione to estradiol at the time indicated by the arrow? A) Granulosa B) Luteal G) Stromal D) Theca externa E) Theca interna CorrectAnswer A. The first half of the menstrual cycle; the follicular phase: which varies in length, begins with menses. During menses, follicle-stimulating hormone (FSH) and luteinizing hormone (LH) concentrations increase and stimulate the developing follicle. Androstenedione is converted to estrone and estradiol via aromatase in the granulosa cells of the follicle. The estrogen then secreted from the granulosa cell is responsible for follicle growth and endometrial proliferation. As estrogen rises, a surge occurs, which in turn stimulates a surge in LH that causes ovulation. Immediately following ovulation, the luteal phase begins as the corpus luteum forms. The corpus luteum secretes progesterone to maintain the endometrial lining. However if no implantation occurs, the corpus luteum degrades to the corpus albicans, and estrogen and progesterone levels decrease, causing menstruation and minor increases in FSH and LH. Incorrect Answers B, C, D, and E. Luteal cells (Choice B) are present in the corpus luteum and are derived from the granulosa cells of the pre-ovu atory follicle. They secrete progesterone and estrogen. However they do not develop until after ovulation (14 days prior to menstruation). Stromal cells (Choice C) are the connective tissue and supporting cells of the ovary. They do not secrete estradiol. Theca externa cells (Choice D) are the cells that form the outer layer of a developing follicle. The theca externa is primarily loose connective tissue and therefore the cells are generally fibroblasts, macrophages, and smooth muscle; these cells do not secrete hormones. Theca interna cells (Choice E) are cells of the follicle that are responsible for generating androstenedione from cholesterol, after which the androstenedione is transported to the granulosa cell to be converted to estradiol. Educational Objective: Granu- osa cells in the developing follicle are responsible for converting androgens received from the theca interna ceils into estradiol via aromatase. e Previous Next Score Report Lab Values Calculator Help Pause
  • 6. Exam Section 1: Item 6 of 50 National Board of Medical Examiners^ Comprehensive Basic Science Self-Assessment Y 6. A 14-year-old girl is brought to the physician by her parents because of a 1-month history of a rash that appears with sun exposure. Her parents tell the physician that she has been eating little food. Physical examination shows a pruritic rash on the exposed areas of the body. Her serum tryptophan concentration is decreased. Urine studies show increased excretion of amino acids, predominantly alanine: isoleucine: leucine, phenylalanine, tryptophan, and valine. Production of which of the following vitamins is most likely impaired in this patient? A) Niacin B) Vitamin B1 (thiamine) C ) Vitamin B2 (riboflavin) D } Vitamin B5 (pantothenic acid) E) Vitamin C Correct Answer: A. Hartnup disease is an autosomal recessive disorder involving a defect in a kidney and intestinal neutral amino acid transporter protein. This defect leads to aminoaciduria and a decreased absorption of neutral amino acids from the gastrointestinal tract resulting in deficiencies of neutral amino acids. Neutral amino acids include tryptophan, phenylalanine, glycine, alanine, vafine: isoleucine, eucine: methionine, and proline. Tryptophan is converted to niacin, so a deficiency in tryptophan can result in niacin deficiency. Niacin deficiency is characterized by rash, glossitis, diarrhea, and neuropsychological disturbances such as dementia and hallucinations. Incorrect Answers: B, C, D, and E. Vitamin B-| (thiamine) (Choice B) is a cofactor for several enzymes in glucose metabolism and adenosine triphosphate production, including pyruvate dehydrogenase and a-ketoglutarate dehydrogenase. Deficiency is characterized by Wernicke encephalopathy, a triad of confusion, ophthalmoplegia, and ataxia. Wernicke encephalopathy is theoretically reversible with administration of high-dose thiamine; if untreated, it can progress to Korsakoff syndrome which is characterized by dementia, confabulation, hallucinations, and psychosis. Vitamin B2 (riboflavin) (Choice C) deficiency is characterized by inflammation and cracking of skin around the lips, mouth, and tongue. It is not associated with aminoaciduria or Hartnup disease Vitamin B5 (pantothenic acid) (Choice D) deficiency is characterized by dermatitis, enteritis, alopecia, and adrenal insufficiency, it is not associated with aminoaciduria or Haitnup disease Vitamin C (Choice E) is found in fruits and vegetables and is necessary far collagen synthesis, iron absorption, immune function and conversion of dopamine to norepinephrine. Deficiency causes scurvy, which is characterized by swollen gums, bruising and poor wound healing, petechiae, perifollicular and subperiosteal hemorrhages, and short, fragile, curly hair. Educational Objective: Hartnup disease is an autosomal recessive disorder involving a defect in a kidney and intestinal neutral amino acid transporter, leading to deficiencies in neutral amino acids such as tryptophan. Tryptophan is converted to niacin, so a deficiency in tryptophan can result in niacin deficiency. Niacin deficiency is characterized by rash, glossitis, diarrhea, and neuropsychological disturbances such as dementia and hallucinations. It Previous Next Score Report Lab Values Calculator Help Pause
  • 7. Exam Section t: Item 7 of 50 National Board of Medical Examiners^ Comprehensive Basic Science Self-Assessment Y 7. Aminoglycoside antibiotics are used for their synergistic action against bacteria, in combination with other agents. These antibiotics demonstrate in vitro synergy for several bacterial species when combined with which of the following classes of antibiotics? A ) Fluoroquinolones B) Macrolides C ) Penicillins D ) Rifamycins E) Tetracyclines Correct Answer: C. Aminoglycosides include gentamicin, neomycin, amikacin, tobramycin: and streptomycin. Their bactericidal function comes from the inhibition of the 3GS subunit of the bacterial ribosome, which precludes protein synthesis by causing misreading of mRNA. Aminoglycosides work synergistically with penicillins, meaning that the combined effect of the two classes is stronger than the effect of either class alone. The penicillin group of antibiotics includes penicillinase-sensitive penicillins (penicillin G; ampicillin, amoxicillin) penicillinase-resistant penicillins (oxacillin, nafcHlin, dicEoxaciflin), and anti-pseudomonal penicillins (ticarcillin, piperacillin). These all inhibit peptidoglycan cross-Einking of the bacterial wall. Aminoglycosides also demonstrate synergistic activity with monobactams such as aztreonam, which also target peptidoglycan cross-linking function. However they do require oxygen for their uptake info the bacterial cell so are ineffective against anaerobes. Bacteria may also develop resistance 1o this class of antibiotics due to the inactivation of the drug by bacterial transferase enzymes, which slightly modify the aminoglycoside structure through acetylation or phosphorylation. Aminoglycoside usage may be complicated by nephrotoxicity, ototoxicity, or neuromuscular damage. They should not be used during pregnancy as they are also a teratogen. Incorrect Answers: A. B( D. and E. Fluoroquinolones {Choice A) inciude ciprofloxacin, levofloxacin, and moxifloxacin. They inhibit prokaryotic DNAgyrase (also known as topoisomerase). Potential side effects include vascular damage, cartilage damage, tendonitis, or tendon rupture. They do network synergistically with aminoglycosides. Macrolides (Choice B) include antibiotics such as azithromycin and erythromycin. These antibiotics also inhibit protein synthesis but instead by inhibiting the SOS subunit of the ribosome. Use of macrolides with aminoglycosides would not provide additional therapeutic benefit particularly given that these classes of drugs have similar mechanisms of action. Rifamycins (Choice D) inhibit the RNA polymerase required to transcribe bacterial DMA. They do not lead to synergistic effects when used with aminoglycosides. Tetracyclines (Choice E) include tetracycline, doxycycline, and minocycline. Like aminoglycosides, these interfere with the 30S subunit of the ribosome. However, rather than causing misreading of mRNA, they prevent attachment of the aminoacyl-tRNA. They do not lead to a synergistic effect when used with aminoglycosides. Educational Objective: Aminoglycosides include gentamicin, neomycin, amikacin, tobramycin, and streptomycin. They inhibit the 3DS subunit of the bacterial ribosome. When used with peptidoglycan linking antibiotics such as the penicillin class, they lead to a synergistic effect and result in improved bacterial killing. © © © © f* Previous Next Score Report Lab Values Calculator Help Pause
  • 8. Exam Section 1: Item 8 of 50 National Board of Medical Examiners^ Comprehensive Basic Science Self-Assessment 8. A 16-year-old boy with moderate intellectual disability is brought to the physician for a routine examination. There is a family history of mild and moderate intellectual disability in his mother and brother, respectively. Physical examination shows a long face prominent ears, and moderately enlarged testicles. Which of the following best describes the genetic mechanism of this patient's disorder? A ) Mutation in a mitochondrial gene B) Presence of an extra sex chromosome C) Translocation of a portion of an aufosome D) Trinucleotide repeat mutation on theX chromosome E) Trisomy of an autosome Correct Answer: D. This patients constellation of findings, including intellectual disab: rty, an elongated face: large ears: and macro-orchidism is suggestive of Fragile X syndrome. Patients may also present with hyperextensible joints and a high-arched palate. Fragile X syndrome is a common cause of inherited intellectual disability and is inherited an X-linked dominant fashion. It is caused by a CGG-trinucleotide repeat expansion within the FMR1 gene. Patients with Fragile X syndrome are at increased risk of mitral valve prolapse and educational difficulties. They are also often diagnosed with autism. As with all trinucleotide repeat expansions, genetic anticipation is seen, wherein future generations have increased severity and/or earlier onset of disease. Incorrect Answers: A, B. C, and E. Mutation in a mitochondrial gene (Choice A) is observed in neurologic and muscular diseases such as Leber hereditary optic neuropathy, maternally inherited diabetes and deafness. myodonic epilepsy with ragged red fibers, and mitochondria! encephalopathy, lactic acidosis, and stroke-like episodes (MELAS) syndrome. Presence of an extra sex chromosome {Choice B) s observed in Klinefelter syndrome, in which patients have an XXY karyotype. Characteristic features include intellectual disability, eunuchoid body shape, tall stature, elongated extremities, and hypogonadism. Translocation of a portion of an autosome (Choice C) is observed in Robertsonian translocations and causes a small portion of cases of Down syndrome. Characteristic physical findings of Down syndrome include intellectual disability, broad, flat, facial features with prominent epicanthal folds, and a single palmar crease. Patients with Down syndrome are at increased risk of Alzheimer dementia, acute lymphoblastic and acute myeloid leukemia, cardiac septal defects, duodenal atresia, and Hirschsprung disease. Trisomy of an autosome (Choice E) is observed in Down syndrome (trisomy 21), Edwards syndrome (trisomy 18), and Patau syndrome (trisomy 13). Edwards syndrome presents with characteristic physical features including intellectual disability, a prominent occiput, low-set ears, micrognathia, clenched, overlapping fingers, and feet with a prominent calcaneus and convexly rounded soles. Patau syndrome presents with characteristic physical features including intellectual disability, deft lip and palate, holoprosencephaly, microphthalmia, cutis aplasia, feet with a prominent calcaneus and convexly rounded soles, and polydactyly. Educational Objective: Fragile X syndrome is an X- inked dominant cause of inherited intellectual disability that presents due to a CGG-trinucleotide repeat expansion in the FMR1 gene, it presents with clinical features including intellectual disability, an elongated face with a prominent jaw. a high-arched palate, large ears, hyperextensible joints, and postpubertal macro-orchidism. © © © 0* & Previous Next Score Report Lab Values Calculator Help Pause
  • 9. Exam Section 1: Item 9 of 50 National Board of Medical Examiners^ Comprehensive Basic Science Self-Assessment Y 9. A 75-year-old woman comes to the physician because of a 3-month history of an enlarging lesion on her face. Physical examination shows a 1.5-cm, brown-black, mottled, scaly lesion with irregular borders. Microscopic examination of a biopsy specimen of the lesion shows atypical melanocytes spread along the basilar layer of the epidermis. Which of the following is the most Eikefy cause of these findings? A ) Acanthosis nigricans B) Actinic keratosis C ) Compound nevus D) Lentigo maligna E) Seborrheic keratosis Correct Answer: D. Malignant melanoma is likely to be present when a lesion demonstrates asymmetry, irregular appearing borders, variable coloration, a diameter greater than 6 mm, and rapid evolution in characteristics. Malignant melanoma could rapidly invade and metastasize, which carries a poor prognosis when diagnosed late. Subtypes include superficial spreading, nodular, lentigo maligna, and acral lentiginous The lentigo maligna type is classically seen in elderly individuals in areas of extensive sun damage, such as the face. Lentigo maligna typically grows slowly and superficial along the dermal-epidermal junction. Any lesion with features suggestive of malignant melanoma should be surgically excised with negative margins and pathologically examined for the depth of derma * invasion. Incorrect Answers: A. B, C. and E. Acanthosis nigricans {Choice A) is characterized by hyperpigmented, velvety patches seen on the neck, upper back, breasts, and axillae which is a marker of metabolic syndrome and diabetes. It is neither pre-malignant nor malignant and there are no atypical cells on histopathologic examination. Actinic keratosis (Choice B) is a premalignant lesion that may progress to squamous cell carcinoma. Clinically, lesions typically appear as light pink, ill-defined macules with a gritty texture in areas of prolonged sun exposure, such as the face, ears, and dorsal hands. Compound nevus (Choice C) is a benign proliferation of melanocytes located in both the epidermis and dermis They are very common. Compound nevi should not display asymmetry, border irregularity, or multiple colors. Development of compound and other benign nevi should cease in the fourth to fifth decade. New or changing nevi after this time are concerning for melanoma. Seborrheic keratosis (Choice E) is a benign proliferation of the epidermis; ' esions exhibit a greasy, adherent appearance. While seborrheic keratoses are often brown, this is due to the keratin produced by the epidermis rather than melanin. Educational Objective: Lentigo maligna is a subtype of malignant melanoma that most commonly manifests on the sun-exposed skin of elderly patients. It is often slow growing and typically confined fo the dermal-epidermal junction. © © © © f* Previous Next Score Report Lab Values Calculator Help Pause
  • 10. Exam Section 1: Item 10 ol 50 National Board of Medical Examiners^ Comprehensive Basic Science Self-Assessment Y 10. A physician wishes to determine the proportion of newborns delivered at a local hospital who had a diagnosis of congenital heart disease within the past year. Which of the following statistical measurements best describes these data? A ) Attributable risk B) Incidence C ) Odds ratio D) Prevalence E) Relative risk Correct Answer: B. incidence is an important epidemiological measure that assesses the rate of occurrence of new disease in a population at-risk. Incidence is the number of new cases expressed as a percentage of the total population at risk over a specified period. In this case: the incidence would be the number of newborns with congenital heart disease divided by the total number of newborns in the study sample within the past year. For example: if 20 newborns are diagnosed with congenital heart disease out of a total of 1000 newborns delivered at the local hospital within the past year, the incidence would be 2%. Incorrect Answers: A, C, D. and E. Attributable risk (Choice A) is a representation of the amount of risk that is associated with a particular exposure. Formally, its definition is the incidence rate in the exposed group minus the incidence rate in the control group. For example, if over the course of a year in a group of 100 smokers there are five myocardial infarctions (incidence rate of 5%), and in a group of 100 non-smokers there are two myocardial infarctions (incidence rate of 2%), the risk attributable to smoking would be 5% minus 2%, or 3%. Odds ratio (Choice C) is a comparison of the odds of an outcome occurring in the exposed group with the odds of that outcome occurring iin a nonexposed control group. Using the example of smokers and non-smokers referenced in Choice A, the odds of myocardial infarction in the smoking group is 5/95 = 0.053. The odds of myocardial infarction in the non-smoking group is 2/93 = 0.0204. The odds ratio would simply be the odds of myocardial infarction in the smoking group divided by the odds of myocardial infarction in the non-smoking group ((5/95)/{2/93)) = 2.53. Prevalence (Choice D) is calculated as the ratio of the number of people with a disease divided by the total number of at-risk persons in a population at a particular point in time. This is also known as point prevalence or disease frequency. For example, if a survey was conducted of a population of 1000 persons and 100 of these individuals were identified as having heart disease, the point prevalence of heart disease would be 100/1000 = 0.10: or 10%. Relative risk (Choice E} compares the risk of an outcome in one group with the risk of an outcome in another group and is often used in cohort studies. It is similar to an odds ratio and can be confused with an odds ratio. The odds ratio can be used to approximate relative risk when the disease or outcome state is rare. An example calculation of relative risk is as follows. In the described population of smokers referenced in Choices A and C, the risk of myocardial infarction in the non-smoking group is 2/100 = 0.020, or 2%. The risk of myocardial infarction in the smoking group is 5/100 = 0.05, or 5%. The relative risk is, therefore, ((5/100) /(2/100)) = 2.50, meaning that the risk of myocardial infarction in the smoking group is 2.5 times the risk in the non-smoking group. Since, in this example, myocardial Infarction is rare, the small disease assumption is valid, and the odds ratio approximates the relative risk (2.58-2.50). Educational Objective: Incidence is a measure of the rate of occurrence of new disease in a population at-risk., which is distinct from prevalence, the measure of the current amount of disease burden in a population. Odds ratio is distinct from relative risk, although if disease burden is rare, the odds ratio may be used to approximate relative risk. © © © Previous Next Score Report Lab Values Calculator Help Pause
  • 11. Exam Section t: Item 11 of 50 National Board of Medical Examiners^ Comprehensive Basic Science Self-Assessment Y 11 A 25-year-old woman comes to the physician after her blood pressure was found to be 130/105 mm Hg at a health fair. She takes no medications. There is no family history of hypertension. Her last menstrual period was 1 week ago. Her blood pressure today is 130/110 mm Hg. Bilateral abdominal bruits are heard Treatment with an angiotensin-converting enzyme (ACE) inhibitor will most likely have which of the following acute effects on this patient's renal function? A ) Decreased concentrating ability secondary to renal angioedema B) Decreased glomerular filtration rate secondary to dilation of efferent arterioles C ) Decreased renal blood flow secondary to dilation of afferent arterioles D } increased concentrating ability secondary to a change in permeability of the collecting duct E) Interstitial nephritis secondary to allergic drug reaction Correct Answer: B. Fibromusctilar dysplasia of the renal artery is the most common cause of renal artery stenosis in younger and middle-aged women. Fibromuscular dysplasia is a non-inflammatory and non-atheroscierotrc angiopathy of medium-sized arteries (eg, renal, carotid) that results in multifocal fibrous and muscular thickening of the arterial wali. which can lead to stenosis. Renal artery stenosis is a cause of secondary hypertension due to abnormal stimulation of the juxtaglomerular apparatus from low afferent blood flow leading to excessive production of renin and angiotensin. The reduced afferent blood flow can result in progressive renal atrophy. Secondary hypertension should be considered in new-onset or treatment-resistant hypertension, or in younger, otherwise healthy patients. ACE inhibitors are a first-line treatment for hypertension as they block the conversion of angiotensin to angiotensin SI, which has direct vasoconstrictive effects as well as promotes salt and water retention. ACE inhibitors may result in a transient acute decrease in glomerular filtration rate (GFR) secondary to dilation of efferent arterioles. This effect is more pronounced in patients with renal artery stenosis, as the baseline reduced afferent blood flow leaves the nephron dependent on efferent arteriole vasoconstriction (mediated by angiotensin 11) to maintain adequate filtration pressure across the glomerulus. ACE inhibitors block this effect. Incorrect Answers: A. C, D, and E. Decreased concentrating ability secondary to renal angioedema (Choice A) is incorrect. ACE inhibitors are associated with angioedema as an adverse effect due to increased bradykinin levels, which may result in swelling of the face, lips, tongue, upper airway, and gastrointestinal tract. Decreased renal blood flow secondary to dilation of afferent arterioles (Choice C) is incorrect as dilation of the afferent arterioles would result in increased renal blood flow. Increased concentrating ability secondary to a change in permeability of the collecting duct (Choice D) occurs with anti-diuretic hormone (ADH) re:ease from the pituitary or exogenous administration of ADH analogs. ADH results In increased aquaporin expression on the luminal surface of collecting duct cells which increases the membrane permeability to water. Interstitial nephritis secondary to allergic drug reaction (Choice E) is a possibility with many medications, but is commonly associated with sulfa-based diuretics, nonsteroidal anti-inflammatory medications, antibiotics, proton pump inhibitors, and rifampin. Patients classically present with fever, hematuria, eosinophiluria, rash, and arthralgias. Educational Objective: ACE inhibitors should be used with caution in patients with renal artery stenosis, as reduced efferent arteriole vasoconstriction may result in a decreased GFR. © © © o o Previous Next Score Report Lab Values Calculator Help Pause
  • 12. Exam Section 1: Item 12 ol 50 National Board of Medical Examiners^ Comprehensive Basic Science Self-Assessment Y 12. A 16-year-old boy is brought to the physician because of a 3-month history of shortness of breath while playing sports. He has no shortness of breath at rest. He says, ''Whenever I run around i cough, so I don't want to be on the basketball team anymore. " He takes no medications and has no known allergies. There is a family history of hypertension and asthma. He is 165 cm (5 ft 5 in) taf! and weighs 63 kg (150 lb); BMI is 25 kg/m2 His respirations are l2/min; and blood pressure is 115/75 mm Hg. Cardiac examination shows no abnormalities except for a midsystolic click at the apex. The lungs are clear to auscultation of the chest. Which of the following best explains this patient's symptoms? A ) Deconditioning B) Exercise-induced asthma C ) Malingering D ) Mitral valve prolapse E) Thyroid disease Correct Answer: B. Exercise-induced asthma most likely accounts for this patients exertional dyspnea. Asthma is characterized by reversible obstruction of the bronchi secondary to hyperreactivity and airway inflammation. Patients present with episodes of wheezing: dry cough, and dyspnea occurring during or shortly after exercise, relieved after rest or the use of bronchodilators. Physical examination during an exacerbation often reveals tachycardia, tachypnea, diffuse wheezes (or rhonchi); and prolonged expiration relative to inspiration. Decreased tactile fremitus may be noted due to air trapping which decreases lung density (leading to reduced transmission of vibrations through the lung parenchyma to the body wall). Treatment is usually with a short acting bronchodilator (SABA) immediately before exercise: although in patients with concomitant asthma not related to exercise: treatment is directed by the severity of underlying asthma, if patients do not tolerate SABAs, montelukast is an alternative option. Incorrect Answers: A. C, D, and E. Deconditioning (Choice A) could cause dyspnea, or the subjective experience of shortness of breath, but it should not cause a cough. It would also be unlikely in a young person with a normal BMI who has previously participated in sports without difficulty. Malingering (Choice C) is defined by falsification of symptoms to obtain a secondary gain. This patient's symptoms have another possible explanation, exercise-induced asthma, and there is no clear secondary gain that he might obtain. Mitral valve prolapse (MVP) (Choice D) is less likely to explain this patient's symptoms than exercise-induced asthma. The mid-systolic click of MVP is often, but not always: followed by a systolic murmur of mitral regurgitation (MR) when symptomatic and causing cardiogenic pulmonary edema. Patients with congenital MVP often have physical findings including scoliosis, pectus excavatum: and low BMI. Symptoms of MVP (if sympfomatic) are more likely to include chest pain. palpitations: and lightheadedness in addition to dyspnea. Thyroid disease (Choice E) is unlikely in this patient, as other associated findings should be present. Dyspnea can result from hyperthyroidism or thyroid storm, but these conditions generally present with diaphoresis, weight loss, exophthalmos, and tremor. Thyroid disease is also less common in male patients of this age group. Educational Objective: Exercise-induced asthma presents with dyspnea, cough: and/or wheezing that begins during or shortly after exercise. It is usually treated with SABAs although montelukast is an alternative therapy. Previous Next Score Report Lab Values Calculator Help Pause
  • 13. Exam Section 1: Item 13 of 50 National Board of Medical Examiners^ Comprehensive Basic Science Self-Assessment Y 13. A 25-year-old man comes to the physician because of a 3-day history of pain and swelling of his right leg. He has no history of major medical illness or recent trauma. Examination of the right lower extremity shows edema and tenderness. Duplex uftrasonography of the right lower extremity shows a thrombus extending into the superficial femoral vein. Further studies show protein C deficiency. Inactivation of which of the following coagulation factors is most likely as a result of this deficiency in this patient? A ) Factors V (proaccelerin) and VIII (antihemophilic factor} B) Factors V (proaccelerin) and IX (plasma thromboplastin component) C ) Factors V (proaccelerin) and XI (plasma thromboplastin antecedent) D ) Factors VIII (antihemophilic factor) and IX (plasma thromboplastin component) E) Factors VIIII (antihemophilic factor) and XI (plasma thromboplastin antecedent) F) Factors IX (plasma thromboplastin component) and XI (plasma thromboplastin antecedent) Correct Ans1 wer: A. Protein C inactivates factors V and VIII; deficiency of protein C leads to a hypercoagulable state predisposing to the development of both venous and arterial thrombi. The coagulation cascade consists of the intrinsic extrinsic, and common pathways. The intrinsic pathway consists of sequential activation of factors XIl: XI. IV, and VIII, and activity of this pathway is measured by the partial thromboplastin time. The extrinsic pathway involves the activation of factor VII and is measured by the prothrombin time or the INFL Both the intrinsic and extrinsic pathways can trigger the common pathway via activation of factor X to Xa. Activated factor X (factor Xa) has several functions, but one is to modify factor V and allow it to form a prothrombinase compfex with factor Xa. This complex helps to form a fibrin clot. Factor Xa is also necessary for the conversion of prothrombin to thrombin. Thrombin binds to thrombomodulin on the surface of endothelial cells inducing a conformational change that allows it to activate protein C: while protein C localizes to the endothelium by binding to the endothelial protein C receptor (EPCR). Activated protein C binds to the surface of activated platelets and degrades factors Va. and Villa, which is the means by which it exerts its anticoagulant effects. Incorrect Answers: B. C, D: E; and F. Factors V and IX (Choice B) and Factors V and XI (Choice C) are incorrect as factors IX and XI are not under inhibitory control by protein C. Activation of factor IX by Xla is an important step in the coagulation cascade and deficiency of factor IX is associated with hemophilia. FactorXI activates factor IX in the intrinsic pathway. Factors VIII and IX (Choice D). when activated, both take part in the intrinsic clotting cascade. While factor VI11 is under inhibitory control by protein C, factor IX is not. Factors VIII and XI (Choice E) are incorrect as factor VIII is under inhibitory control by protein C but factor XI is not. Factors IX and XI (Choice F) are both activated irr the intrinsic dotting cascade and serve to accelerate clotting. They are not influenced directly by protein C. Educational Objective: Protein C is a natural anticoagulant that is activated by thrombin after thrombin binds to the endothelial surface. Activated protein C binds to the surfaces of activated platelets and degrades factors Va and Villa, thereby exerting negative feedback on the clotting cascade. © © © A o o Previous Next Score Report Lab Values Calculator Help Pause
  • 14. Exam Section 1: Item 14 ol 50 National Board of Medical Examiners^ Comprehensive Basic Science Self-Assessment Y 14. Which of the following is most directly responsible for concentrating testosterone in the lumen of the seminiferous tubules? A ) Androgen-binding protein B) Follicle-stimulating hormone (FSH) C ) FSH/gonadotropin-releasing hormone D) InhJbrn E) Luteinizing hormone Correct Answer: A. Androgen-binding protein (ABP) is produced by the Sertoli cells of the seminiferous tubules via the regulation of follicle-stimulating hormone (FSH). Testosterone is produced by Leydig ceils in the interstitium adjacent to the seminiferous tubules; the production of which is regulated by of luteinizing lormone (LH). Once released into the lumen of the seminiferous tubuies; ABP facilitates spermatogenesis by binding to testosterone, allowing this otherwise lipophilic hormone to concentrate in the lumen. Normal spermatogenesis requires high local concentrations of luminal testosterone. Incorrect Answers: B, C, D; and E. Follicle-stimulating hormone (FSH) (Choice B) is produced by gonadotropic cells in the anterior pituitary. It plays an important role in spermatogenesis by stimulating Sertoli cells to produce ABP and by directly stimulating sperm development. Its role in the concentration of luminal testosterone lis indirect and mediated by ABP FSH/gonadotropin-releasing hormone (Choice C) is produced by the hypothalamus and stimulates the production and release of FSH and LH from gonadotropic cells in the anterior pituitary. Its role in the concentration of testosterone in the lumen of the seminiferous tubuies is indirect and is mediated by both FSH and ABP. Inhibin (Choice D) is produced by Sertoli cells and exerts negative feedback on gonadotropic cells in the anterior pituitary to regulate the production of FSH. Luteinizing hormone (Choice E) is also produced by gonadotropic cells in the anterior pituitary. It is important for stimu ating the production of testosterone but is not involved in the process of concentrating testosterone in the lumen of the seminiferous tubules. Educational Objective: Androgen-binding protein (ABP) is produced by the Sertoli cells of the seminiferous tubules under the direction of foilic: e-stimulating hormone. Once released into the lumen of the seminiferous tubules, ABP facilitates spermatogenesis by binding to testosterone and concentrating it in the lumen of the tubules. 0 0 # Previous Next Score Report Lab Values Calculator Help Pause
  • 15. Exam Section 1: Item 15 ol 50 National Board of Medical Examiners^ Comprehensive Basic Science Self-Assessment 15. A 23-year-old woman has had the lesions shown in her mouth for 3 days. She has had frequent similar episodes over the past 15 years. The lesions are exacerbated by spicy, salty, and acidic food and drinks. They last approximately 1 week and resolve spontaneously. Visits to the dentist seem to trigger the development of the sores. Which of the following is the most likely diagnosis? A) Aphthous ulcers 6) Candidiasis C) Geographic tongue D ) Koplik spots E) Leukoplakia F) Lichen planus G) Psoriasis Correct Answer: A. Aphthous ulcers are painful, round to oval, shallow oral ulcers. Tney are the most common cause of mouth sores and can be idiopathic or related to underlying conditions such as lupus erythematosus (in which case they are not painful) or Behget syndrome. They may demonstrate pathergy, which is the development of new erosions at the site of a trauma, such as after a dental procedure. When these lesions develop recurrently a diagnosis of recurrent aphthous stomatitis is made. It is commonly seen in adolescence and young adulthood, and episodes typically decrease with increasing age. The etiology is multifactoriat: but the lesions can be exacerbated by spicy acidic, or salty foods, as in this case. Stress can also lead to exacerbations. Recurrent aphthous stomatitis may be treated by optimizing oral hygiene, avoiding exacerbating factors, and treating pain with topical anesthetics and coating agents. ncorrect Answers: B. C. D. E. F. and G. Oral candidiasis (Choice B) demonstrates thick, white plaques on the tongue or buccal mucosa, which can be scraped off with a tongue blade. It is commonly seen in immunosuppressed individuals, such as those with poorly controlled HIV infection, or patients using a steroid inhaler and altering the normal oral microbiome. Geographic tongue (Choice C) is a feature of psoriasis and often seen in chi dren. The tongue demonstrates a maze-like pattern of white, linear patches. Ulcers are not a typical feature. Koplik spots (Choice D) are bright red macules with a bluish-white center on the buccal mucosa, which are a sign of an active measles infection. Koplik spots are accompanied by a prodromal fever, cough, coryza, conjunctivitis, and a confluent maculopapular rash that starts at the headfneck and spreads to the trunk, excluding the palms and soles. Leukoplakia (Choice E) refers to the development of white plaques in the mouth, which cannot be scraped off by a tongue blade, and are typically seen on the tongue or buccal mucosa. It may be due to an underlying Epstein-Barr virus infection and is common in patients with HIV infection or malignancy. Oral lichen planus (Choice F) is characterized by white patches with a stellate appearance on the buccal and gingival mucosa. Erosions can also occur, but the white, stellate patches will also be present, unlike in this case. Psoriasis (Choice G) may affect the oral mucosa in the form of geographic tongue. However, it is more classically characterized by thick, salmon-colored plaques with si:very-white scale on the extensor extremities. Educational Objective: Aphthous ulcers are painful, round to oval, shallow oral ulcers. They are the most common cause of mouth sores and can be exacerbated by certain foods, trauma, or emotiona stress. m ft" in Previous Next Scone Report Lab Values Calculator Help Pause
  • 16. Exam Section 1: Item 16 ol 50 National Board of Medical Examiners^ Comprehensive Basic Science Self-Assessment y 16. A 20-year-old woman comes to the emergency department 30 minutes after slipping on ice and extending her hand to break her fall. Palpation of the anatomic snuff-box produces pain. A wrist x-ray is most likely to show a fracture of which of the following carpal bones? A) Scaphoid B) Lunate C ) Triquetrum D ) Pisiform E) Trapezium F) Trapezoid G) Capitate H ) Hamate Correct Answer: A. Falling onto an outstretched hand can iead to traumatic injuries such as distal radius fractures, elbow dislocations, or fractures of the carpal bones of the wrist. A common pattern of injury with this mechanism is a scaphoid fracture. The scaphoid bone is part of the lateral column of the wrist and supports force transmission from the hand to the lateral aspect of the radius. Fractures of this bone typically present with lateral wrist pain and tenderness in the anatomic snuff-box, which is the dorsal depression between extensor poMicis iongus and abductor pollicis longus. The scaphoid has a blood supply that proceeds from distal to proximal. Displacement of a fracture of this bone may lead to decreased blood supply of the proximal fragment, leading to avascular necrosis and debilitating wrist pain and deformity. Because of this, it is important to identify and appropriately treat scaphoid fractures. Incorrect Answers: B. C, D, E. F, G, and H. Fractures of the lunate (Choice B) are an uncommon injury. More commonly with high energy injuries to the wrist, the capitate and the remaining carpal bones may dislocate from the concave surface of the lunate. This is known as a perilunate dislocation and may lead to acute compression of the median nerve. The triquetrum (Choice C) is a carpal bone located in the ulnar aspect of the wrist. Fractures in this location are uncommon. If fractured, it would present with tenderness along the ulnar aspect of the wrist. The pisiform (Choice D) is a carpal bone in the ulnar aspect of the wrist. It is a pea-shaped sesamoid bone that can be mistaken for a fracture fragment. Fractures of the pisiform are uncommon. The trapezium (Choice E) is the bone at the base of the thumb that can also be palpated in the floor of the anatomic snuff-box. Fracture of the trapezium is uncommon and typically presents with pain after trauma to the thumb. The trapezoid (Choice F) is a wedge-shaped carpal bone just proximal to the second metacarpal. Fracture of the trapezoid is an uncommon carpal injury. The capitate (Choice G) is a larger carpal bone in the center of the wrist. It articulates with the lunate and is dislocated in a perilunate dislocation. The hamate (Choice H) is a carpal bone in the ulnar aspect of the wrist that has a process along its volar surface, referred to as the hook of the hamate, that ads as a covering for the finger flexor tendons of the uJnar digits. Fracture of the hook of the hamate can occur while placing a high load through the wrist while holding a handle (eg, baseball bat, golf club, sledgehammer). This injury can lead to compression of the ulnar nerve in Guyon canal and irritation or impingement of the adjacent tendons. Educational Objective: Fracture of the scaphoid presents with radial wrist pain and tenderness in the anatomic snuff-box, typically following a fall on an outstretched hand. Identification and appropriate treatment of this injury is important as the retrograde blood supply to this bone places it at risk for avascular necrosis of the proximal fragment. O m m rts Previous Next Score Report Lab Values Calculator Help Pause
  • 17. Exam Section 1: Item 17 of 50 National Board of Medical Examiners^ Comprehensive Basic Science Self-Assessment Y 17. A 76-year-old man undergoes laparotomy for resection of an abdominal aortic aneurysm. During the procedure, an incidental finding of acquired coionic diverticula is made. The diverticula in this patient are most likely present in which of the following? A ) Ascending colon B) Cecum C ) Descending colon D) Sigmoid colon E) Transverse colon Correct Answer: D. The sigmoid colon is the most common location for diverticula to form. Diverticula are outpouchings of the mucosal and submucosal layers into the muscular layer that occur at weak points in the gut wall where the small arterioles of the vasa recta penetrate, it is hypothesized that abnormal motility in the colon causes increased intraluminal pressure with subsequent herniation of the mucosa through weak points in the colonic wall. This may happen in the sigmoid coton because the diameter of the sigmoid is smaller than other parts of the colon, so abnormal peristalsis in this area causes higher intraluminal pressure compared to other segments. Risk factors for development of diverticula include obesity and a diet high in red meat and low in fiber. Diverticulosis predisposes to lower gastrointestinal bleeding (Gl). It also predisposes to diverticulitis, a bacterial infection of a diverticulum that leads to a local inflammatory response. Diverticulitis presents classically with fever, left lower quadrant abdominal pain, and occasionally with bloody diarrhea. Incorrect Answers: A, B. C. and E. The ascending colon (Choice A) can be a site of colorectal carcinoma (CRC). Because stool is generally liquid in the ascending colon and tumors in this location tend not to be exophytic C:RC can present [ate as symptoms are less common. While diverticula may occur in the ascending colon, it is a less common location than the sigmoid. Similarly, the transverse colon (Choice E} and descending colon (Choice C) may develop diverticula, but these sites are less common than the sigmoid colon. Cecum (Choice B) is the junction between the ileum and the ascending colon. It is not a common site of diverticuEosis, but since the appendix lies in close proximity to it, the cecum can occasionally become inflamed in severe, acute appendicitis. Educational Objective: Diverficufosis refers to a condition in which the colonic mucosa and submucosa herniate into the muscular layer at weak points where the vasa recta penetrate the colonic wall. Risk factors include chronic constipation and a low fiber diet. The sigmoid colon is the most frequently involved part of the colon as its smaller diameter predisposes to greater intraluminal pressure during peristalsis, thereby increasing the risk for diverticula formation. * It Previous Next Score Report Lab Values Calculator Help Pause
  • 18. Exam Section 1: Item 13 ol 50 National Board of Medical Examiners^ Comprehensive Basic Science Self-Assessment Y 18. During a period of 36 hours, an 80-year-old woman has increasingly severe abdominal pain followed by fever chills, tachycardia, hypotension and. finally, shock. Blood cultures grow Escherichia coii Her condition worsens and. despite supportive therapy and antibiotics; she dies 4 days after the onset of the illness. Which of the following is the most likely cause of the initial hypotension? A) Excessive production of nitric oxide B) Generation of hydrogen peroxide G) Hemorrhage D } Induction of endothelial adhesion molecuEes E) Platelet aggregation Correct Answer: A. Sepsis is a systemic inflammatory syndrome that results from a dysregulated and exaggerated immune response to an infection. Sepsis can be complicated by shock and multiorgan failure with a high mortality rate. Septic shock is characterized by an impaired response of the vasculature to vasoconstricting stimuli with markedly decreased systemic vascular resistance, tachycardia: increased cardiac output, oliguria; and lactic acidosis. Excessive production of nitric oxide is associated with hypotension in the setting of sepsis. Inducible nitric oxide synthetase is a nitric oxide-producing enzyme that is upregulated through tyrosine kinase activation in response to proinflammatory cytokines and binding by Hpopolysaccharides. Nitric oxide activates guanyiate cyclase in vascular smooth muscle resulting in muscle relaxation and vasodilation because of increased intracellular cyclic guanosine monophosphate concentration. Incorrect Answers: B, C: D: and E. Generation of hydrogen peroxide (Choice B) occurs in phagolysosomes by the enzyme superoxide dismutase: which utilizes free oxygen radicals produced by NADPH oxidase. Excess free radical production is associated with host tissue injury. Hemorrhage (Choice C) may result in shock secondary to hypovolemia and decreased oxygen carrying capacity of the blood secondary to loss of hemoglobin. This patient's abdominal pain is likely the result of an infectious enteritis, colitis, or peritonitis, and is less likely from a ruptured abdominal aortic aneurysm, which may cause shock from internal exsanguination. Induction of endothelial adhesion molecules (Choice D) is a key step in the recruitment and migration of leukocytes to sites of infection and injury. The action of nitric oxide on the vascular smooth muscle results in hypotension, whereas the expression of endothelial adhesion molecuEes is involved in the immune response to localized infection. Platelet aggregation (Choice E) and activation occurs in response to inflammation and helps promote the innate immune response, although it does not directly cause the patient's hypotension. Diffuse platelet aggregation may result in thrombocytopenia in the setting of sepsis. Educational Objective: Sepsis is a dysregulated systemic inflammatory syndrome that may occur in response to infection. It may progress to hypotension and septic shock. Excessive nitric oxide production causing diffuse vasodilation is one of the mechanisms of hypotension in sepsis. O 0 © 0 0 Previous Next Score Report Lab Values Calculator Help Pause
  • 19. Exam Section 1: Item 19 of 50 National Board of Medical Examiners^ Comprehensive Basic Science Self-Assessment Y 19. A G5-year-old woman comes to the physician because of a 3-month history of headache, weakness of her arms, and left flank pain: she also has had a 14-kg (31-lb) weight loss during this period. Physical examination shows weakness of the proximal upper and lower extremity muscles. There is augmentation of strength with repetitive testing of the deltoid muscles. An MRI of the brain shows a single well-demarcated mass surrounded by edema in the right frontal lobe. A stereotactic biopsy specimen of the :esion shows a malignant small blue cell neoplasm that expresses cytokeratin, chromogranirv and synaptophysin. Which of the following is the most likely diagnosis? A ) Anaplastic ependymoma B) Extranodal primary central nervous system lymphoma C ) Glioblastoma multiforme D ) Primary cerebral neuroblastoma E) Pulmonary smalt cel: carcinoma metastatic to the brain Correct Answer: E The patient's presentation is most consistent with pulmonary smali cell carcinoma metastatic to the brain. Pulmonary small cell tumors are typically centrally located in the lungs and associated with tobacco use. They are neoplasms of neuroendocrine ceils and may be associated with numerous paraneoplastic syndromes, inctuding Cushing syndrome due to adrenocorticotropic hormone production, syndrome of inappropriate antidiuretic hormone, Lambert-Eaton myasthenic syndrome due to presynaptic calcium channel antibody production, and paraneoplastic myelitis, encephalitis, and subacute cerebellar degeneration. Proximal extremity weakness and augmentation of strength with repetitive testing of the deltoid muscles are suggestive of Lambert-Eaton myasthenic syndrome. Histologic features of pulmonary small cell carcinoma include small dark blue tumor cells lacking nucleoli with a high nuclear to cytoplasm ratio. The brain is a common site for metastatic disease. Incorrect Answers: A. B, C. and 0. Anaplastic ependymoma (Choice A) is a central nervous system neoplasm formed from ependymal cells. Location typically involves the fourth ventricle. Histologic characteristics include perivascular pseudorosettes formed by malignant cells arranged around a blood vessel. Extranodal primary central nervous system lymphoma (Choice B) is a rare type of malignant non-Hadgkin lymphoma, often associated with an underlying immunodeficiency syndrome (eg, AIDS). Histologic examination may reveal large, atypical lymphocytes. Glioblastoma multiforme (Choice C) is a malignant primary brain tumor characterized by central necrosis on histology. Imaging features include an expansile mass crossing the corpus callosum with surrounding vasogenic edema. Seizures are a common presenting symptom, as are headaches and focal neurologic deficits. Primary cerebral neuroblastoma (Choice D) is a malignancy of neuroendocrine cells associated with sympathetic nervous tissue. Neuroblastomas may arise from the adrenal glands; the sympathetic chain, or the central nervous system, and may secrete catecholamines. Metastatic pulmonary small cell carcinoma is more common in adults than a primary cerebral neuroblastoma. Educational Objective: Solitary brain lesions may be secondary to a primary central nervous system malignancy: metastatic disease, infection or abscess. Histologic analysis and clinical features can help narrow the diagnosis. Pulmonary small cell carcinomas are neuroendocrine tumors associated with paraneoplastic syndromes and brain metastases. © © © © f* Previous Next Score Report Lab Values Calculator Help Pause
  • 20. Exam Section 1: Item 20 ol 50 National Board of Medical Examiners^ Comprehensive Basic Science Self-Assessment Y 20. Failure of normal differentiation of the endoderm in the embryonic lung bud is most likely to affect the development of which of the following? A ) Capillary patterns B) Cartilage in bronchi C) Smooth muscle on the bronchi D) Surfactant secretion E) Tracheal rings Correct Answer: D. Endoderm is one of the three primary embryonic germ layers and composes the innermost layer of the early developing organism. Endoderm derivatives include the epithelial linings of the respiratory tract, gastrointestinal tract biliary system, genitourinary tract, vagina, and middle ear. Organs that arise from the endoderm include the liver, parathyroid glands,, thymus, pancreas, and the follicular and parafollicular cells of the thyroid. Surfactant secretion in the mature lung is achieved by type II pneumocytes, a component of the respiratory epithelium. Defective differentiation of the endoderm in the embryonic lung bud would most ikely result in impaired development of type II pneumocytes and reduced secretion of surfactant. Incorrect Answers: A, B. C, and E. Capillary patterns (Choice A), cartilage in bronchi (Choice B), smooth muscle on the bronchi (Choice C), and tracheal rings (Choice b} are all derivatives of the mesoderm. Mesoderm is the middle embryonic germ layer and primarily responsible for development of connective tissue structures, including muscle, dermis, bone, cartilage, dura mater, the cardiovascular system, lymphatic system, blood components, kidneys, adrenal cortex, and reproductive organs. Educationa Objective: Ceils of the respiratory epithelium arise from the embryonic endoderm germ layer. This includes type I pneumocytes. which form the simple squamous epithelium of the alveoli, and type II pneumocytes, which secrete surfactant. 0 0 & % 0* Previous Next Score Report Lab Values Calculator Help Pause
  • 21. Exam Section 1: Item 21 ol 50 National Board of Medical Examiners^ Comprehensive Basic Science Self-Assessment Y 21. A G7-year-old woman comes to the physician for a health maintenance examination. Her brother and mother have a history of colon cancer. The physician recommends colonoscopy: taut the patient says that she would prefer only for her stool to be tested for blood. The physician explains that testing the stool for occult blood is not appropriate in this case. The physician is most likely concerned about which of the following regarding this test? A) Low sensitivity B) Low specificity C ) Potential for a false-positive result D } Uncertain negative predictive value E) Uncertain positive predictive value Correct Answer: A. Sensitivity is the ability of a test to detect a disease if it is present. A test is described as sensitive if it has a high likelihood of disease detection, and therefore a low likelihood of false negativity. High sensitivity is therefore useful in ruling out a disease. This is because a negative result from a high sensitivity test indicates a low likelihood that the disease is present. Because of this, high sensitivity tests are useful for screening in which proving the absence of disease and limiting the number of false negative results are of utmost importance. In the case of cancer screening, a highly sensitive test allows the clinician to be confident that a negative test means that the patient is disease-free. In contrast, a test with poor sensitivity, if negative, does not provide strong evidence or confidence that the patient does not have the disease. In the case described, the fecal occult blood test demonstrates both a low sensitivity and specificity and is of limited use to the physician and the patient for ruling out cancer. incorrect Answers: B, C: D: and E. Low specificity (Choice B) describes a test that is subject to false-positive errors, meaning that a positive test does not have a high likelihood of disease. High specificity is required to confirm a diagnosis. An example of high specificity testing for colon cancer would be a colonoscopy with biopsy of a lesion with staining and molecular testing under microscopy. Direct visualization of malignant cells on microscopy is highly specific for a cancer diagnosis, that is, there is a low likelihood of a false cancer diagnosis. Potential for a false-positive result (Choice C) is possible with a fecal occult blood test. For example, a bleeding hemorrhoid or taking an iron supplement can cause a positive fecal occult blood test. This is unlikely, however, to cause concern by the physician in this scenario as a false positive test does not necessarily put the patient at risk for an undiagnosed colorectal cancer, although it may lead to unnecessary further diagnostic examinations. Missing an early cancer diagnosis due to poor test sensitivity could lead to early mortality and is a more pressing issue. Uncertain negative predictive value (Choice D) is not of immediate concern in this scenario. Negative predictive value is based on both the test sensitivity and the pretest probability of the patient having the disease. Although certain individuals have higher or lower risk of colon cancer based on lifestyle and genetics, pretest probability and alterations in the negative predictive value do not alter the need for a sensitive test for screening purposes. Uncertain positive predictive value (Choice E) is also not an immediate concern. Positive predictive value is based on both test specificity and the pretest probability of disease. Positive predictive value is the likelihood that a person has a disease, given a positive test. A positive predictive value is of greater importance in confirmatory testing. Educational Objective: High sensitivity tests are required for effective disease screening. High specificity tests are necessary for confirmation of the disease. Negative and positive predictive values are functions not only of sensitivity and specificity, but also of the pretest probability of the disease. O 0 © Previous Next Score Report Lab Values Calculator Help Pause
  • 22. Exam Section 1: Item 22 ol 50 National Board of Medical Examiners^ Comprehensive Basic Science Self-Assessment Y 22. A 5-year-old boy is brought to the emergency department after ingesting 10 oz of a household cleaning solvent. He is treated for acute hepatic and renal failure for 1 week and then discharged. During the next month; regeneration of this boy's mature hepatocytes and renal tubular epithelial cells will be accomplished mostly by which of the following mechanisms? A ) Activation of stem cells to enter G1 phase of the cell cycle B) Decreased apoptosis at GrMtransition of the cell cycle C ) Recruitment of cells from G0 into the cell cycle D } Shortened time for progression of cells through the cell cycle E) Terminal differentiation by cells exiting from the cell cycle Correct Answer: C. When hepatocytes or renal tubular epithelial cells are destroyed, the remaining cells are recruited from quiescence (the phase) to re-enter the cell cycle. In healthy patients, the majority of hepatocytes and renal tubular epithelial cells are in the GQ phase. When cells are destroyed such as in acute liver or renal failure, genes are induced that prime remaining ceils to re-enter the cell cycle from quiescence. These cells transition from GQ phase to G1 phase, where growth factors engender cell growth. Ceils that grow sufficiently surpass the restriction point, at which point they are committed to DNA replication and cell division via mitosis. After the G-j restriction point, the cells transition to the S phase: when DNA replicates. The cells enter another growth phase: the G2 phase, and finally the M phase, when mitosis occurs and hepatocytes and renal tubular epithelial cells regenerate. Incorrect Answers: A. Bf D. and E. Activation of stem cells to enter the G1 phase of the cell cycle (Choice A) does not play a major role in the regeneration of hepatocytes or renal tubular epithelial cells. Stem cells from the bone marrow or within the liver/kidney itself may minorly contribute to regeneration, but the recruitment of the large population of quiescent cells into the cell cycle is more crucial. Decreased apoptosis at G ^ -M transition of the cell cycle (Choice B) and shortened time for progression of cells through the cell cycle (Choice D) would not lead to regeneration of hepatocytes or renal tubular epithelial cells. Most cells are quiescent (not in the cell cycle) at baseline so decreasing apoptosis or shortening the cell cycle time would not lead to an appreciable increase in cells. Terminal differentiation by cells exiting from the cell cycle (Choice E) does not occur after hepatic or renal damage and would prevent cells' future ability to regenerate. Many cell types (eg, skeletal muscle cells) terminally differentiate and lose their ability to regenerate. Educational Objective: The vast majority of hepatocytes and renal tubular epithelial cells are in the GQ phase (quiescence) at baseline. When cells are destroyed, the remaining cells re-enter the cell cycle at the G1 phase to grow and divide, leading to regeneration. 0 0 ^ Previous Next Score Report Lab Values Calculator Help Pause
  • 23. Exam Section 1: Item 23 of 50 National Board of Medical Examiners^ Comprehensive Basic Science Self-Assessment Y 23. A 17-year-old boy is brought to the physician by his mother because she is concerned that his puberty is delayed. The mother states: "He is so short. His father is 6 feet 5 inches tall. I don't understand why he has not had his growth spurt." When the mother leaves the room, the patient states: "I'm fine. ! don't know what's the matter with her. She wants me to be tall like my dad." The patient is 175 cm (5 ft 9 in) tall and weighs 70 kg (155 lb); BMI is 23 kg/m2 Sexual development is Tanner stage 4. in addition to reassuring the mother that her son is fine, which of the following is the most appropriate initial statement by the physician to the mother? A ) "Since your son is fine with his height you should try to accept him as he is. " B) "Tel: me more about your concerns about your son's height. " C ) "We’ll do some blood tests just to be sure that all your son's hormone levels are okay." D ) "Your son is average for his height and weight." E) "Your son is not going to be any taller." Correct Answer: B. When patients or patients' family members express medical concems: physicians should initially ask open-ended questions to explore the understanding and fears of the patient or family member. The physician can then tailor further discussion and reassurances to address these knowledge gaps and fears. Asking open-ended questions also invites the family member to elaborate on their concerns, as there may be medically or psychfatrically relevant details that the family member reveals on further discussion. Further, listening to the specific concerns of the patient or family members will improve therapeutic alliance. Incorrect Answers: A. C; D, and E. Giving parental advice (Choice A) would be unwarranted in this situation and outside of the physician's scope of practice. If a parent's behavior is clearly affecting the menta or physical health of the patient, the physician may tactfully bring the issue to the parent's attention. However this patient's health is not clearly impacted by his mother's concern. Physicians should refrain from ordering tests that are medically unnecessary based on patient or family concern (Choice C). The physician should instead reassure and educate after listening to the patient or family's specific concerns. Saying that the patient is average for his height and weight (Choice D) would not address this mother's concern about the patient being shorter than his father Tnis statement would also prevent elaboration of the mother's specific concerns, understanding, and fears. Informing the patient's mother that her son is not going to be any taller (Choice E) would not be accurate or reassuring. This statement would also prevent elaboration of the mother’s specific concerns, understanding, and fears. Educational Objective: When patients or families express medical concerns physicians should ask open-ended questions to elucidate the specific nature of the concern and the patient's or family member's understanding. The physician can then tailor further discussion to address knowledge gaps and specific fears, and may learn additional, medically relevant details about the concern. © © © Previous Next Score Report Lab Values Calculator Help Pause
  • 24. Exam Section 1: Item 24 ol 50 National Board of Medical Examiners^ Comprehensive Basic Science Self-Assessment y 24. A 34-year-old woman is admitted to the hospital for treatment of pulmonary tuberculosis. Infliximab therapy was initiated G months ago for severe Crohn disease. This pharmacotherapy most likely inhibited which of the following immunologic functions in this patient? A ) Activation of nuclear factor KB to induce expression of interleukin-10 (IL-10) B) Direct toxicity to the causal organism C ) Maintenance of granulomas D } Recruitment of segmented neutrophils to ingest and kill the bacteria E) Stimulation of B lymphocytes to produce neutralizing antibodies against the causal organism Correct Answer: C. Tumor necrosis factor-a (TNF-a) is a cytokine secreted by macrophages, which supports granuloma formation. Granulomas are collections of histiocytes, or macrophages with abundant pink cytoplasm that often contain multi-nucleated giant cells and are surrounded by lymphocytes One purpose of granuloma formation is to sequester an infection or foreign body. Monoclonal antibody therapy targeted against TNF-a increases a patients risk for active mycobacterial infection because it stops the production of TNF-a, leading to breakdown of the granuloma and release of any contained organism. Monoclonal antibodies against TNF-a that are specifically used to treat Crohn disease include infliximab, adalimumab, and cerfclizumab. These medications are known to increase the risk of reactivating latent Mycobacterium tuberculosis because of their deleterious effect on granuloma formation and maintenance. All patients who are considered for treatment with these agents must undergo screening for latent M tuberculosis infection. If positive, treatment for latent M. tuberculosis with nine months of isoniazid is indicated. Incorrect Answers: A. B, D. and E. Activation of nuclear factor KB to induce expression of interleukin-10 (IL-10) {Choice A) occurs within the Th2 cell type. 1L-1G attenuates the immune response by inhibiting activated macrophages and decreasing expression of Th1 cytokines. Inhibition of this IL-1D would therefore stimulate the immune process against infectious organisms, not decrease it. TNF- a is a cytokine that maintains granulomas and assists in walling off infections such as tuberculosis. It does not have any direct toxicity to the causal organism (Choice B),and thus this is not the mechanism by which anti-TNF-a monoclonal antibodies increase the risk of latent tuberculosis reactivation. IL-8, not TNF-a, is responsible for the recruitment of segmented neutrophils to ingest and kill the bacteria (Choice D). This process is not affected by the inhibition of TNF-a. Rituximab, an airti-CD20 monoclonal antibody, diminishes B lymphocytes and is used in chemotherapy regimens for lymphoma. It is also used in the treatment of autoimmune conditions including vasculitis, rheumatoid arthritis, and pemphigus vulgaris. This monoclonal antibody, not a TNF-a inhibitor, prevents stimulation of B lymphocytes to produce neutralizing antibodies against the causal organism (Choice E). Educational Objective: The use of monoclonal antibody therapy against TNF-a is associated with an increased risk of reactivated latent M. tuberculosis secondary to inadequate granuloma maintenance. Because of this, all prospective candidates for this therapy should be screened for latent M. tuberculosis infection and accordingly treated if positive. © © © © f* Previous Next Score Report Lab Values Calculator Help Pause
  • 25. Exam Section 1: Item 25 ol 50 National Board of Metlicai Examiners^ Comprehensive Basic Science Self-Assessment y 25. An 8-year-old boy continues to bleed excessively after tooth extraction. Prothrombin time, bleeding time, and platelet count are within the reference range. Partial thromboplastin time is prolonged but corrects after addition to the assay chamber of plasma from a patient with hemophilia A. Which of the following is the most likely diagnosis? A ) Acute disseminated intravascular coagulation B) Factor V (proaccelenn) deficiency C ) Factor Vt! (proconvertin) deficiency D ) Hemophilia A E) Hemophilia B F) Immune thrombocytopenic purpura G) von Willebrand disease Correct Answer: E Hemophilia B is an X-linked bleeding disorder caused by absent, decreased, or dysfunctional factor IX and most likely explains this patient's prolonged bleeding and increased partial thromboplastin time (PIT). As patients with hemophilia A lack factor VIII but have normal levels of factor IXf addition of plasma from a patient with hemophilia A would correct the PTT as demonstrated m this patient with hemophilia B. Factor IX is a component of the intrinsic clotting cascade and serves to activate factor X to Xa. which subsequently converts prothrombin to thrombin and facilitates the formation of a fibrin clot. The activity of the coagulation factors in the intrinsic coagulation cascade is measured by the PTT while the activity of the extrinsic pathway is measured by the prothrombin time (PT)f which is normal in this patient. The clinical severity of hemophilia 6 is variable. Patients with severe disease present early in life with easy bruising, bleeding following a minor procedure, or hemarthrosis. Patients with less severe disease may not present until they experience an event such as trauma or surgery Treatment includes replacement of the deficient factor. Incorrect Answers: A. R, C. D, F, and G. Acute disseminated intravascular coagulation (Choice A) is a syndrome characterized by overwhelming activation of the clotting cascade often precipitated by malignancy, sepsis, or obstetrical emergencies. Endothelial dysfunction leads to the formation of microthrombi and depletion of coagulation factors. Microthrombi cause shearing stress on erythrocytes leading to microangiopathic hemolytic anemia, while the depletion of coagulation factors manifests as a prolonged FT and PTT and increases the risk of major bleeding. Factor V (proaccelerin) deficiency (Choice B) is a rare inherited bleeding disorder that may present with mucocutaneous bleeding or major bleeding following trauma or surgery. Factor V is required as a cofactor for the formation of thrombin in the common pathway of the coagulation cascade. Deficiency is treated with fresh frozen plasma (FFP)f which contains factor V. Factor VII (proconvertin) deficiency (Choice C) Is a rare bleeding disorder with a spectrum of clinical severity Patients who are most affected present with heavy menstrual bleeding or bleeding following invasive procedures. Treatment is with factor replacement, prothrombin complex concentrate, or FFP. Hemophilia A (Choice D) is an X-linked bleeding disorder that presents similarly to hemophilia B and is caused by an absent or reduced level of factor VIII. Addition of plasma from another patient with hemophilia A would not correct the PTT. Immune thrombocytopenic purpura (Choice F) is caused by circulating antibodies against platelets that leads to thrombocytopenia. The PT and PTT are normal Bleeding, if it occurs, tends to be mucocutaneous. von Willebrand disease (Choice G) is one of the most common hereditary bleeding disorders and is due to quantitative or qualitative abnormality of von Wil ebrand factor, which binds platelets and subendothekai collagen in primary hemostasis. Impaired platelet adherence leads to a prolonged bleeding time. It can present with epistaxis, gingival bleeding, petechiae, easy bruising, and menorrhagia. Educational Objective: Hemophilia B is an X-linked bleeding disorder that is caused by a deficiency in factor IX leading to an increased PTT. Symptoms depend on severity but include prolonged bleeding following invasive procedures or trauma, easy bruising and hemarthrosis. Treatment is with recombinant factor IX. o m m Previous Next Score Report Lab Values Calculator Help Pause
  • 26. Exam Section 1: Item 26 ol 50 National Board of Metlicai Examiners^ Comprehensive Basic Science Self-Assessment y 26. The diagram shows the major factors that determine blood pressure. Which of the fallowing labeled factors is affected most by an aradrenergtc antagonist? ^Centralnervous system ' i ® Peripheral resistance (arteriolar ) 1=fCardiac output j x Blood pressure © ^Stroke volume ] fHeart rate CD "Contractility j £Venous return " ^ Blood volume! Capacitance vessel tone ' (venolar) © A ) B) C ) D) El Correct Answer: C. The autonomic nervous system plays a primary role in the maintenance of blood pressure via its effects on peripheral arteriolar resistance: heart rate, myocardial contractility: venous capacitance, and to an indirect degree., blood volume. The sympathetic nervous system primarily acts through a- and (3-adrenergic receptors, which are stimulated by dopamine, epinephrine: and norepinephrine. In relation to blood pressure regulation, the a t receptor leads to smooth muscle contraction especially of the vasculature, which increases blood pressure by increasing systemic vascular resistance. The p;receptor in comparison, causes increases iin heart rate, myocardiat contractility, and renin release, which all lead to increases in blood pressure. An antagonist at the a1 receptor would decrease vascular smooth muscle contraction to a greater extent around arteries, leading to vascular dilation and decreased blood pressure as mean arterial pressure is directly related to cardiac output and systemic vascular resistance. Incorrect Answers: A. Ei. D, and E. Central nervous system inputs to cardiac output (Choice A) can occur through the limbic system in response to strong emotions or anticipation of physical activity or through central components of the autonomic nervous system (eg. brainstem, spinal autonomic preganglionic neurons). These autonomic stimuli to the heart would be less affected by an a1 receptor antagonist than peripheral resistance, as the stimuli would also include effects on the p,receptor By contrast, central a2-adrenoreceptors are the target of the agonist clonidine, which results in diminished sympathetic tone,, forming the basis for the use of clonidine in hypertensive emergency. Heart rate (Choice B) is primarily affected by the pi receptor, with agonism leading to increases in heart rate. As part of the parasympathetic nervous system, heart rate is decreased by M2 receptors. a1 receptor antagonists do not affect heart rate directly, though can cause reflex tachycardia. Blood volume (Choice D) is affected by changes in red blood cells and plasma volume, such that anemia or hemorrhage would decrease blood volume. However changes in plasma volume can also be mediated by diuretics (decreasing plasma volume) or renin release (increasing plasma volume). a1 receptors do not moderate blood volume. Capacitance vessel tone (Choice E} describes the ability of the vessel to hold a volume of blood at a specific blood pressure. The capacity of the venular compartment increases with decreased somatic muscle movement, valvular dysfunction, and nitroglycerin administration. Venule tone decreases with ,receptor blockade, but to a lesser extent than periphera arteriolar resistance as there is minimal vascular smooth muscle in venules compared to arterioles. Educational Objective: The autonomic nervous system plays a primary role in the maintenance of blood pressure via its effects on peripheral arteriolar resistance, heart rate, myocardial contractility, venous capacitance, and to an indirect degree, blood volume. aradrenergic receptors primarily modulate mean arterial- pressure by increasing peripheral arteriolar resistance. © © m - "s 7 Previous Next Scone Report Lab Values Calculator Help Pause
  • 27. Exam Section 1: Item 27 of 50 National Board of Metlicaf Examiners^ Comprehensive Basic Science Self-Assessment y -Y' NT VI V 5 iVft 3! V EZI aVL V2 v? 3IJ PW VS R ! /I : VI I 1 v__._. J II U. _l 'V . . .l~A. ... VS 27. A previously healthy 21-year-old woman comes to the office because of a 2-month history of shortness of breath and fatigue. Her most recent menstrual period was 3 months ago. Menses previously had occurred at regular 28-day intervals. She tells the physician that she thinks she may be pregnant. She takes no medications and has not seen a physician for several years. She appears healthy. She is 160 cm (5 ft 3 in) tall and weighs 54 kg (120 lb); BMI is 21 kg/m2 Vital signs are within normal limits. The lungs are dear Cardiac examination shows a normal S1; a widely split S2 that does not change with respiration, and a grade 3/6 holosystoMc murmur that is loudest at the lower left sternal border and radiates to the upper left sternal border EGG is shown. The most likely cause of these findings is dysfunction of which of the following structures? A) Atrial septum B) Ductus arteriosus C ) interventricular septum D ) Pulmonic valve E) Tricuspid valve Correct Answer: A. Atrial septal defect is a common congenital malformation of the interatrial septum The most common type is an ostium secundum defect although ostium primum defects are associated with trisomy 21. The atrial septal defect results in a left-to-right shunt with abnormal flow of blood from the left atrium to the right atrium, resulting in relative volume overload of the right atrium and ventricle. This increased stroke volume of the right ventricle results in delayed closure of the pulmonic valve, which presents as a fixed, split S2l and low-grade physiologic ejection murmur on cardiac auscultation. The increased right heart volumes also result in a prominent right ventricular impulse on physical exam and may present an increased risk for the development of a right bundle branch block, which may be present on ECG as seen in this case. If the atrial septal defect remains uncorrected, it can result in the development of Eisenmenger syndrome secondary to prolonged pulmonary vasculature remodeling resulting in pulmonary arterial hypertension and shunt reversal leading to cyanosis. Asymptomatic atrial septal defects may become clinically significant in the setting of increased blood flow (such as during pregnancy). Incorrect Answers: B, C: D. and E. A patent ductus arteriosus (Choice B) is a persistent extracardiac conduit between the aorta and the pulmonary artery that has failed to obliterate after birth. It results in a continuous, machine-like murmur best heard in the left second intercostal space, radiating to the clavicle. Defects of the interventricular septum (Choice C) are characterized by a hoiosystolic murmur best heard in the left lower sternal border They do not classically result in a fixed, split S2 increased flow across the pulmonic valve (Choice D) due to shunting of blood from the left atrium to the right atrium results in the tixed S2 associated with an atrial septal defect. The pulmonic valve itself is typically norma!. Tricuspid valve (Choice E) dysfunction results in tricuspid regurgitation, which presents as a hoiosystolic murmur best heard in the left lower sternal border. Educational Objective: A fixed, widely split S2 is characteristic of an atrial septal defect due to increased blood flow through the pulmonic valve. Severe defects can result in pulmonary hypertension and development of Eisenmenger syndrome overtime, with reversal of the left to right shunt. o © 0 * 0 * K Previous Next Score Report Lab Values Calculator Help Pause
  • 28. Exam Section 1: Item 28 ol 50 National Board of Metlicai Examiners^ Comprehensive Basic Science Self-Assessment y MJW _: 4 * m i « - V > I LQJI. fc * L ' .i i •HI " T^W. JP TP r 1 *1 ? .£ p . L s'm - f i “ - —1 ' A . >CJ * -c fc “VO i. ..I p r SC" fi J p Ol IT * r £ T m a .-i - E 1 i a p tr h. d k ^p «J- » R a I i a . a. £ "a J -K 28. A 53-year-old man has had progressive difficulty swallowing for the past 3 months. He has a 10-year history of heartburn with esophageal regurgitation of gastric contents. Tissue obtained on biopsy of the lower third of the esophagus is shown. Which of the following best describes the nature of this lesion? A } Basal zone hyperplasia of submucosal glands B) Intestinal metaplasia of squamous epithelium C ) Malignant transformation of epithelium into squamous carcinoma D ) Squamous metaplasia of submucosal glands Correct Answer: B. Jntestina1 metaplasia of squamous epithelium in the esophagus, also known as Barrett esophagus, can be a consequence of prolonged gastroesophageai reflux disease (GERD): which occurs when acidic gastric contents reflux backward through the lower esophageal sphincter into the esophagus. The mucosa of the esophagus is comprised of squamous epithefium and does not traditionally encounter such an acidic environment. Constant exposure to acidic intraluminal contents induces a change in cell type from squamous epithelium to the columnar glandular epithelium found in the intestines as an adaptive response. These metaplastic cells will exhibit a brush border and goblet cells. Metaplasia can eventually lead to dysplasia, which is premalignant. Patients with confirmed Barrett esophagus should be evaluated at regular intervals determined by the presence and/or grade of dysplasia. Treatment involves ablation of the dysplastic cells via endoscopy and management of the underlying GERD with a proton pump inhibitor, dietary modification, and smoking cessation. Incorrect Answers: A, C; and D. Basal zone hyperplasia of submucosal glands (Choice A) is not the pathologic change observed in Barrett esophagus, although submucosal gland secretions do neutralize acidic luminai contents. They also lubricate the esophagus which allows for the food bolus to pass. Malignant transformation of epithelium into squamous carcinoma (Choice C) occurs with esophageal squamous carcinoma, which is more common in patients who consume alcohol and smoke cigarettes. Barrett esophagus primarify predisposes to adenocarcinoma, not to squamous carcinoma. Squamous metaplasia of submucosal glands (Choice D) is also associated with the development of esophageal adenocarcinoma. Submucosal glands contain progenitor cells that may play a role in the pathogenesis of dysplasia as these progenitor cells serve as a source of potentially dysplastic or neoplastic cells, however, the pathophysiology of Barrett esophagus involves intestinal metaplasia. Educational Objective: Barrett esophagus develops in individuals with chronic GERD and is histologically characterized by intestinal metaplasia whereby the normal squamous epithelium is replaced by columnar epithelium. Overtime,, dysplasia can develop, predisposing to esophageal adenocarcinoma. O 0 m (E T Previous Next Scone Report Lab Values Calculator Help Pause
  • 29. Exam Section 1: Item 29 of 50 National Board of Medical Examiners^ Comprehensive Basic Science Self-Assessment Y 29. A 50-year-old man comes to the emergency department because of a 2-week history of progressive shortness of breath. His pulse is 90/min, respirations are 26/mrn. and blood pressure is 120/80 mm Hg. Physical examination shows no other abnormalities. Laboratory studies show: Arterial Pco2 Arterial Po2 Arterial 02content Mixed venous Po2 Mixed venous 02content 8 vol% (N=10%-16%) 30 mm Hg 96 mm Hg 12 vol% (N=17%-21%) 36 mm Hg Which of the following is the most likety explanation for these findings? A) Anemia B) Drug-induced alveolar hypoventilation C ) Residence at a high altitude D ) Severe regional mismatching of alveolar ventilation and pulmonary capillary perfusion E) Voluntary hyperventillation Correct Answer: A. The differentia! for dyspnea is broad and encompasses a range of disorders that involve impaired delivery of oxygen to tissue and/or reduced elimination of carbon dioxide from the body. The laboratory studies in this case indicates a reduced arterial and venous oxygen content. The oxygen content of the blood is a function of the oxygen carrying capacity (essentially the hemoglobin concentration), percent saturation of hemoglobin, and partial pressure of dissolved molecular oxygen (Po ^ The equation to compute oxygen content is thus: Oxygen content = 1.34*[Hemaglobin]*(Arterial Oxygen Saturation) + G.003*(Arterial PoJ. The amount of dissolved oxygen is negligible compared to the oxygen transported by hemoglobin. The patient in this case has reduced oxygen content in the arterial and mixed venous circulation with a normal arterial Po2 In the absence of a hemoglobinopathy (eg. methemoglobinemia, carboxyhemoglobinemia), the patient's oxygen saturation is expected to be normal. The most likely diagnosis is anemia with a decreased hemoglobin concentration. The arteriai Pco2 is decreased indicating hyperventilation, which is expected in the setting of decreased oxygen delivery to tissue. incorrect Answers: B, C: D: and E. Drug-induced alveolar hypoventilation (Choice B) would result in an increased arterial Pco ^ Potential etiologies include central nervous system depressants such as opioid analgesics and benzodiazepines. Residence at a high altitude (Choice C) would be expected to result in a decreased Po2 with adaptive changes that maintain an adequate oxygen carrying capacity. These changes include secondary erythrocytosis with increased hemoglobin concentration and increased levels of 2,3-bisphosphoglyceric acid: which stabilizes the deoxygenated state of hemoglobin and promotes increased oxygen reiease to tissue. Severe regional mismatching of alveolar ventilation and pulmonary capillary perfusion (Choice D) occurs when either ventilation or perfusion to a region of lung is impaired. An example is pulmonary embolism, in which a region of ventilated lung has obstructed blood flow. Voluntary hyperventilation (Choice E) results in a respiratory alkalosis, with a decreased Pco ^ . Symptoms include dizziness, weakness, and syncope. The oxygen carrying capacity of the blood would not be reduced in the absence of other factors Educational Objective: Delivery of oxygen to tissue is largely dependent on the hemoglobin concentration as it is the primary transporter of oxygen in the blood. Other contributing factors include the oxygen saturation of hemoglobin, and, less Significantly, the partial pressure of dissolved molecular oxygen. O 0 Previous Next Scone Report Lab Values Calculator Help Pause
  • 30. Exam Section 1: Item 30 ol 50 National Board of Medical Examiners^ Comprehensive Basic Science Self-Assessment Y 30. An investigator is studying the effects of triiodothyronine (TJ and thyroxine (TJin hepatocytes in an experimental animal model. Which of the following best describes the action of these thyroid hormones on this target tissue? A ) Both T3 and T4 bind to the melanocortin 2 receptor on the cell surface B) Both T5 and T4 enter the nucleus C ) T3 is converted to I 4 in the cytosol D ) Thyroid hormone receptors preferentially bind T4 over T3 Correct Answer: B. Both T3 (triiodothyronine) and T4 (thyroxine) are hormones that act on nuclear receptors, requiring them to enter the target ceil to exert effects. Unlike other lipophilic hormones, thyroid hormones contain charged amino acids that prevent passive diffusion across the cellular membrane and thus enter by facilitated diffusion. Thyroid hormone transporters transport both T3 and T4 into the cell to reach their receptors. The thyroid hormone receptors are nuclear receptors that contain DNA-binding domains. Nuclear receptors can initially be in either the cytosol or nucleus. Once nuclear receptors bind their respective hormones: they translocate into the nucleus, if not already there, where they act as DNA transcription factors to regulate the expression of target genes. Incorrect Answers: A. C; and D. Binding to the melanocortin 2 receptor on the cell surface (Choice A) does not occur with either T3 orT4. The melanocortin 2 receptor is also known as the adrenocorticotropic hormone (AGTH) receptor and is specific for ACTH. This receptor is a G protein-coupled receptor and does not actively transport ACTH inside the cell. Conversion of T3 to T4 in the cytosoi (Choice C) does not occur. T4 is the less active form of thyroid hormone and is converted to T3 In target cells. Once in the cell nucleus, T3 preferentially binds the receptor with greater affinity than T4 (Choice D): although both hormones are capable of binding and activating the receptor. Educational Objective: T3 and T4 act on nuclear receptors, requiring them to enter the target cell to exert effects. Unlike other lipophilic hormones, thyroid hormones contain charged amino acids that prevent passive diffusion across the cellular membrane. Thyroid hormone transporters transport both T3 and T4 into the cell to reach their receptors. 0 0 # 0 f* Previous Next Score Report Lab Values Calculator Help Pause